Pythagorean Theorem Exercises

Pythagorean theorem practice problems with answers.

There are eight (8) problems here about the Pythagorean Theorem for you to work on. When you do something a lot, you get better at it. Let’s get started!

Here’s the Pythagorean Theorem formula for your quick reference.

Pythagorean Theorem - a squared plus b squared is equal to c squared

Note: drawings not to scale

Problem 1: Find the value of [latex]x[/latex] in the right triangle.

the legs of a right triangle are 6 and 8

[latex]10[/latex]

Problem 2: Find the value of [latex]x[/latex] in the right triangle.

the leg of a right triangle is 12 and the hypotenuse is 16

[latex]4\sqrt 7 [/latex]

Problem 3: Find the value of [latex]x[/latex] in the right triangle.

right triangle with legs square root 3 and square root 6

[latex]3[/latex]

Problem 4: The legs of a right triangle are [latex]5[/latex] and [latex]12[/latex]. What is the length of the hypotenuse?

[latex]13[/latex]

Problem 5: The leg of a right triangle is [latex]8[/latex] and its hypotenuse is [latex]17[/latex]. What is the measure of its other leg?

[latex]15[/latex]

Problem 6: Suppose the shorter leg of a right triangle is [latex]\sqrt 2[/latex]. The longer leg is twice the shorter leg. Find the hypotenuse.

[latex]\sqrt {10} [/latex]

Problem 7: The hypotenuse of a right triangle is [latex]4\sqrt 2[/latex]. If the longest leg is half the hypotenuse, what is the length of the shortest leg?

[latex]2\sqrt 6[/latex]

Problem 8: Find the value of [latex]x[/latex] of the right isosceles triangle.

the hypotenuse of a right isosceles triangle is 2 times square root 3

[latex]\sqrt 6[/latex]

You might also like these tutorials:

  • Pythagorean Theorem
  • Pythagorean Triples
  • Generating Pythagorean Triples
  • Maths Questions

Pythagoras Theorem Questions

Class Registration Banner

Pythagoras theorem questions with detailed solutions are given for students to practice and understand the concept. Practising these questions will be a plus point in preparation for examinations. Let us discuss in brief about the Pythagoras theorem.

Pythagoras’ theorem is all about the relation between sides of a right-angled triangle. According to the theorem, the hypotenuse square equals the sum of squares of the perpendicular sides.

= (Perpendicular) + (Base)

Click here to learn the proof of Pythagoras’ Theorem .

Video Lesson on Pythagoras Theorem

pythagorean theorem problem solving questions

Pythagoras Theorem Questions with Solutions

Now that we have learnt about the Pythagoras Theorem, lets apply the same by solving the following questions.

Question 1: In a right-angled triangle, the measures of the perpendicular sides are 6 cm and 11 cm. Find the length of the third side.

Let ΔABC be the triangle, right-angled at B, such that AB and BC are the perpendicular sides. Let AB = 6 cm and BC = 11 cm

Then, by the Pythagoras theorem,

AC 2 = AB 2 + BC 2

\(\begin{array}{l}\Rightarrow AC=\sqrt{(AB^{2}+BC^{2})}=\sqrt{6^{2}+11^{2}}\end{array} \)

\(\begin{array}{l}=\sqrt{36+121}=\sqrt{157}\end{array} \)

∴ AC = √157 cm.

Question 2: A triangle is given whose sides are of length 21 cm, 20 cm and 29 cm. Check whether these are the sides of a right-angled triangle.

If these are the sides of a right-angled triangle, it must satisfy the Pythagoras theorem.

We have to check whether 21 2 + 20 2 = 29 2

Now, 21 2 + 20 2 = 441 + 400 = 841 = 29 2

Thus, the given triangle is a right-angled triangle.

If three integers a, b and c are such that a + b = c , then (a, b, c) is called Pythagorean triples.

For any given integer m, (m – 1, 2m, m + 1) is the Pythagorean triplet.

Learn more about .

Question 3: Find the Pythagorean triplet with whose one number is 6.

Now, m 2 + 1 = 9 + 1 = 10

and m 2 – 1 = 9 – 1 = 8

Therefore, the Pythagorean triplet is (6, 8, 10).

Question 4: The length of the diagonal of a square is 6 cm. Find the sides of the square.

Let ABCD be the square, and let AC be the diagonal of length 6 cm. Then triangle ABC is the right-angled triangle such that AB = BC (∵ all sides of a square are equal)

By Pythagoras theorem,

⇒ AC 2 = 2AB 2

⇒ AC = √2 AB

⇒ AB = (1/√2) AC = (1/√2)6 = 3√2 cm.

Question 5: A ladder is kept at a distance of 15 cm from the wall such that the top of the ladder is at the height of 8 cm from the bottom of the wall. Find the length of the wall.

Let AB be the ladder of length x.

Pythagoras theorem question 5

AC 2 + BC 2 = AB 2

\(\begin{array}{l}\Rightarrow AB=\sqrt{AC^{2}+BC^{2}}\end{array} \)

\(\begin{array}{l}\Rightarrow x=\sqrt{8^{2}+15^{2}}=\sqrt{64+225}\end{array} \)

⇒ x = 17 cm

∴ Length of the ladder is 17 cm.

Question 6: Find the area of a rectangle whose length is 144 cm and the length of the diagonal 145 cm.

Let the rectangle be ABCD

Pythagoras theorem question 6

\(\begin{array}{l}\Rightarrow AD=\sqrt{AC^{2}-CD^{2}}=\sqrt{145^{2}-144^{2)}\end{array} \)

⇒ AD = √(21025 – 20736) = √289

⇒ AD = 17 cm

Thus, area of the rectangle ABCD = 17 × 144 = 2448 cm 2 .

  • Properties of Triangles
  • Congruence of Triangles
  • Similar Triangles
  • Trigonometry

Question 7: A boy travels 24 km towards east from his house, then he turned his left and covers another 10 km. Find out his total displacement?

Let the boy’s house is at point O, then to find the total displacement, we have to find OB.

Pythagoras theorem question 7

Clearly, ΔOAB is a right-angled triangle, by Pythagoras theorem,

\(\begin{array}{l} OB=\sqrt{OA^{2}+AB^{2}}=\sqrt{24^{2}-10^{2}}\end{array} \)

⇒ OB = √(576 + 100) = √676

⇒ OB = 26 km.

Question 8: Find the distance between a tower and a building of height 65 m and 34 m, respectively, such that the distance between their top is 29 m.

The figure below shows the situation. Let x be the distance between the tower and the building.

Pythagoras theorem question 8

In right triangle DCE, by Pythagoras theorem,

CE = √(DE 2 – DC 2 ) = √(29 2 – 21 2 )

⇒ x = √(841 – 441) = √400

⇒ x = 20 m.

∴ the distance between the tower and the building is 20 m.

Question 9: Find the area of the triangle formed by the chord of length 10 cm of the circle whose radius is 13 cm.

Let AB be the chord of the circle with the centre at O such that AB = 10 and OA = OB = 13. Draw a perpendicular OM on AB.

Pythagoras theorem question 9

By the property of circle, perpendicular dropped from the centre of the circle on a chord, bisects the chord.

Then, AM = MB = 5 cm.

Now, in right triangle OMB,

OB 2 = OM 2 + MB 2

⇒ OM = √(OB 2 – MB 2 )

⇒ OM = √(13 2 – 5 2 ) = √(169 – 25)

⇒ OM = √144 = 12 cm

Area of triangle OAB = ½ × AB × OM

= ½ × 10 × 12

= 60 cm 2 .

Question 10: Find the length of tangent PT where P is a point which is at a distance 10 cm from the centre O of the circle of radius 6 cm.

Given, OP = 10 cm and OT = 6m.

We have to find the value of PT.

Pythagoras theorem question 10

By the property of tangents, the radius of the circle is perpendicular to the tangent at the point of contact.

Thus, triangle OTP is a right-angled triangle.

∴ by the Pythagoras theorem,

OP 2 = OT 2. + PT 2

⇒ PT = √(OP 2 – OT 2 ) = √(10 2 – 6 2 )

⇒ PT = √(100 – 36) = √64

⇒ PT = 8 cm.

Related Video on Pythagorean Triples

pythagorean theorem problem solving questions

Practice Questions on Pythagoras Theorem

1. Find the area of a right-angled triangle whose hypotenuse is 13 cm and one of the perpendicular sides is 5 cm.

2. Find the Pythagorean triplet whose one member is 15.

3. Find the perimeter of a rectangle whose diagonal is 5 cm and one of its sides is 4 cm.

4 if a pole of length 65 cm is kept leaning against a wall such that the pole reaches up to a height of 63 cm on the wall from the ground. Find the distance between the pole and the wall.

5. Find the area of the triangle inscribed within a circle of radius 8.5 cm such that one of the sides of the triangle is the diameter of the circle and the length of the other side is 8 cm.

(Hint: The triangle is formed in semi-circular region and angle of a semi-circle is of 90 o )

Learn about various mathematical concepts in a simple manner with detailed information, along with step by step solutions to all questions, only at BYJU’S. Download BYJU’S – The Learning App to get personalised videos.

MATHS Related Links

pythagorean theorem problem solving questions

Register with BYJU'S & Download Free PDFs

Register with byju's & watch live videos.

  • Home   |  
  • About   |  
  • Contact Us   |  
  • Privacy   |  
  • Newsletter   |  
  • Shop   |  
  • 🔍 Search Site
  • Easter Color By Number Sheets
  • Printable Easter Dot to Dot
  • Easter Worksheets for kids
  • Kindergarten
  • All Generated Sheets
  • Place Value Generated Sheets
  • Addition Generated Sheets
  • Subtraction Generated Sheets
  • Multiplication Generated Sheets
  • Division Generated Sheets
  • Money Generated Sheets
  • Negative Numbers Generated Sheets
  • Fraction Generated Sheets
  • Place Value Zones
  • Number Bonds
  • Addition & Subtraction
  • Times Tables
  • Fraction & Percent Zones
  • All Calculators
  • Fraction Calculators
  • Percent calculators
  • Area & Volume Calculators
  • Age Calculator
  • Height Calculator
  • Roman Numeral Calculator
  • Coloring Pages
  • Fun Math Sheets
  • Math Puzzles
  • Mental Math Sheets
  • Online Times Tables
  • Online Addition & Subtraction
  • Math Grab Packs
  • All Math Quizzes
  • 1st Grade Quizzes
  • 2nd Grade Quizzes
  • 3rd Grade Quizzes
  • 4th Grade Quizzes
  • 5th Grade Quizzes
  • 6th Grade Math Quizzes
  • Place Value
  • Rounding Numbers
  • Comparing Numbers
  • Number Lines
  • Prime Numbers
  • Negative Numbers
  • Roman Numerals
  • Subtraction
  • Add & Subtract
  • Multiplication
  • Fraction Worksheets
  • Learning Fractions
  • Fraction Printables
  • Percent Worksheets & Help
  • All Geometry
  • 2d Shapes Worksheets
  • 3d Shapes Worksheets
  • Shape Properties
  • Geometry Cheat Sheets
  • Printable Shapes
  • Coordinates
  • Measurement
  • Math Conversion
  • Statistics Worksheets
  • Bar Graph Worksheets
  • Venn Diagrams
  • All Word Problems
  • Finding all possibilities
  • Logic Problems
  • Ratio Word Problems
  • All UK Maths Sheets
  • Year 1 Maths Worksheets
  • Year 2 Maths Worksheets
  • Year 3 Maths Worksheets
  • Year 4 Maths Worksheets
  • Year 5 Maths Worksheets
  • Year 6 Maths Worksheets
  • All AU Maths Sheets
  • Kindergarten Maths Australia
  • Year 1 Maths Australia
  • Year 2 Maths Australia
  • Year 3 Maths Australia
  • Year 4 Maths Australia
  • Year 5 Maths Australia
  • Meet the Sallies
  • Certificates

Pythagoras Theorem Questions

Welcome to our Pythagoras' Theorem Questions area. Here you will find help, support and questions to help you master Pythagoras' Theorem and apply it.

For full functionality of this site it is necessary to enable JavaScript.

Here are the instructions how to enable JavaScript in your web browser .

Pythagoras' Theorem Questions

Here you will find our support page to help you learn to use and apply Pythagoras' theorem.

Please note: Pythagoras' Theorem is also called the Pythagorean Theorem

There are a range of sheets involving finding missing sides of right triangles, testing right triangles and solving word problems using Pythagoras' theorem.

Using these sheets will help your child to:

  • learn Pythagoras' right triangle theorem;
  • use and apply the theorem in a range of contexts to solve problems.

Pythagoras' Theorem

where a,b and c are the sides of a right triangle.
Side c is the hypotenuse (longest side).

Pythagoras' Theorem - in more detail

Pythagoras' theorem states that in a right triangle (or right-angled triangle) the sum of the squares of the two smaller sides of the triangle is equal to the square of the hypotenuse.

Right triangle labelled

In other words, \[ a^2 + b^2 = c^2 \]

where c is the hypotenuse (the longest side) and a and b are the other sides of the right triangle.

What does this mean?

This means that for any right triangle, the orange square (which is the square made using the longest side) has the same area as the other two blue squares added together.

Right triangle pythagoras

Other formulas that can be deduced from the Pythagorean theorem

As a result of the formula \[ a^2 + b^2 = c^2 \] we can also deduce that:

  • \[ b^2 = c^2 - a^2 \]
  • \[ a^2 = c^2 - b^2 \]
  • \[ c = \sqrt{a^2 + b^2} \]
  • \[ b = \sqrt {c^2 - a^2} \]
  • \[ a = \sqrt {c^2 - b^2} \]

Pythagarean Theorem Examples

Example 1) find the length of the missing side..

Pythagoras theorem example 1

In this example, we need to find the hypotenuse (longest side of a right triangle).

So using pythagoras, the sum of the two smaller squares is equal to the square of the hypotenuse.

This gives us \[ 4^2 + 6^2 = ?^2 \]

So \[ ?^2 = 16 + 36 = 52 \]

This gives us \[ ? = \sqrt {52} = 7.21 \; cm \; to \; 2 \; decimal \; places \]

Example 2) Find the length of the missing side.

Pythagoras example 2

In this example, we need to find the length of the base of the triangle, given the other two sides.

This gives us \[ ?^2 + 5^2 = 8^2 \]

So \[ ?^2 = 8^2 - 5^2 = 64 - 25 = 39 \]

This gives us \[ ? = \sqrt {39} = 6.25 \; cm \; to \; 2 \; decimal \; places \]

Pythagoras' Theorem Question Worksheets

The following questions involve using Pythagoras' theorem to find the missing side of a right triangle.

The first sheet involves finding the hypotenuse only.

A range of different measurement units have been used in the triangles, which are not drawn to scale.

  • Pythagoras Questions Sheet 1
  • PDF version
  • Pythagoras Questions Sheet 2
  • Pythagoras Questions Sheet 3
  • Pythagoras Questions Sheet 4

Pythagoras' Theorem Questions - Testing Right Triangles

The following questions involve using Pythagoras' theorem to find out whether or not a triangle is a right triangle, (whether the triangle has a right angle).

If Pythagoras' theorem is true for the triangle, and c 2 = a 2 + b 2 then the triangle is a right triangle.

If Pythagoras' theorem is false for the triangle, and c 2 = a 2 + b 2 then the triangle is not a right triangle.

  • Pythagoras Triangle Test Sheet 1
  • Pythagoras Triangle Test Sheet 2

Pythagoras' Theorem Questions - Word Problems

The following questions involve using Pythagoras' theorem to solve a range of word problems involving 'real-life' type questions.

On the first sheet, only the hypotenuse needs to be found, given the measurements of the other sides.

Illustrations have been provided to support students solving these word problems.

  • Pythagoras Theorem Word Problems 1
  • Pythagoras Theorem Word Problems 2

Geometry Formulas

  • Geometry Formula Sheet

Here you will find a support page packed with a range of geometric formula.

Included in this page are formula for:

  • areas and volumes of 2d and 3d shapes
  • interior angles of polygons
  • angles of 2d shapes
  • triangle formulas and theorems

This page will provide a useful reference for anyone needing a geometric formula.

Triangle Formulas

Here you will find a support page to help you understand some of the special features that triangles have, particularly right triangles.

Using this support page will help you to:

  • understand the different types and properties of triangles;
  • understand how to find the area of a triangle;
  • know and use Pythagoras' Theorem.

All the free printable geometry worksheets in this section support the Elementary Math Benchmarks.

  • Geometry Formulas Triangles

Here you will find a range of geometry cheat sheets to help you answer a range of geometry questions.

The sheets contain information about angles, types and properties of 2d and 3d shapes, and also common formulas associated with 2d and 3d shapes.

Included in this page are:

  • images of common 2d and 3d shapes;
  • properties of 2d and 3d shapes;
  • formulas involving 2d shapes, such as area and perimeter, pythagoras' theorem, trigonometry laws, etc;
  • formulas involving 3d shapes about volume and surface area.

Using the sheets in this section will help you understand and answer a range of geometry questions.

How to Print or Save these sheets 🖶

Need help with printing or saving? Follow these 3 steps to get your worksheets printed perfectly!

  • How to Print support

Subscribe to Math Salamanders News

Sign up for our newsletter to get free math support delivered to your inbox each month. Plus, get a seasonal math grab pack included for free!

math salamanders news image

  • Newsletter Signup

Return to Geometry Section

Return from Pythagoras Theorem Questions to Math Salamanders Homepage

Math-Salamanders.com

The Math Salamanders hope you enjoy using these free printable Math worksheets and all our other Math games and resources.

We welcome any comments about our site or worksheets on the Facebook comments box at the bottom of every page.

New! Comments

TOP OF PAGE

Follow Me on Pinterest

© 2010-2024 Math Salamanders Limited. All Rights Reserved.

  • Privacy Policy
  • Copyright Policy

High Impact Tutoring Built By Math Experts

Personalized standards-aligned one-on-one math tutoring for schools and districts

In order to access this I need to be confident with:

Powers and roots

Pythagorean Theorem

Here you will learn about the Pythagorean Theorem, including how to find side measurements of a right triangle and using Pythagoras’ theorem to check and see if a triangle has a right angle or not.

Students will first learn about Pythagorean Theorem as part of geometry in 8 th grade and continue to use it in high school.

What is the Pythagorean Theorem?

The Pythagorean Theorem states that the square of the longest side of a right triangle (called the hypotenuse) is equal to the sum of the squares of the other two sides.

Pythagorean Theorem formula shown with triangle ABC is:

a^2+b^2=c^2

US Webpages_ Pythagorean Theorem 1 US

Side c is known as the hypotenuse . The hypotenuse is the longest side of a right triangle. Side a and side b are known as the adjacent sides. They are adjacent, or next to, the right angle.

You can only use the Pythagorean Theorem with right triangles.

For example,

Let’s look at this right triangle:

US Webpages_ Pythagorean Theorem 2 US

Above, three square grids have been drawn next to each of the sides of the triangle.

The area of the side of length 3=3 \times 3=3^2=9

The area of the side of length 4=4 \times 4=4^2=16

The area of the side of length 5=5 \times 5=5^2=25

The sum of the areas of the squares on the two shorter sides is equal to the area of the square on the longest side.

When you square the sides of the two shorter sides of a right triangle and add them together, you get the square of the longest side.

3^2+4^2=5^2

3, 4, 5 is known as a Pythagorean triple.

There are other Pythagorean triples such as 5, 12, 13 and 8, 15, 17.

If you know two lengths of a right triangle, you can use Pythagorean Theorem to work out the length of the third side.

Pythagorean Theorem Proof

Use the drawing of a square with a smaller square shown inside with the proof below.

US Webpages_ Pythagorean Theorem 3 US

The area of each triangle is \cfrac{1}{2} \, a b and the area of the smaller square is c^2.

There are two ways to find the area of the larger square.

  • Combine the area of the four congruent triangles and the smaller square: \begin{aligned}& =\cfrac{1}{2} \, a b+\cfrac{1}{2} \, a b+\cfrac{1}{2} \, a b+\cfrac{1}{2} \, a b+c^2 \\\\ & =2 a b+c^2\end{aligned}
  • Multiply the side lengths of the larger square together: \begin{aligned}& =(a+b)(a+b) \\\\ & =a^2+2 a b+b^2\end{aligned}

Now set the two expressions equal to each other to prove a^2+b^2=c^2 \text{:}

\begin{aligned}a^2 +2 a b+b^2 &=2 a b+c^2 \\\\ -2 a b \hspace{0.3cm} & \hspace{0.3cm} -2 a b \\\\ a^2+b^2&=c^2\end{aligned}

Since the triangles formed by the vertices of a square will also be right triangles, the proof above shows that a^2+b^2=c^2 will always be true for the sides of a right triangle.

[FREE] Pythagorean Theorem Worksheet (Grades 8)

[FREE] Pythagorean Theorem Worksheet (Grades 8)

Use this quiz to check your grade 8 students’ understanding of pythagorean theorem. 15+ questions with answers covering a range of 8th grade topics on pythagorean theorem to identify areas of strength and support!

3D Pythagorean Theorem

You can find the length AG in the cuboid ABCDEFGH using the Pythagorean Theorem.

US Webpages_ Pythagorean Theorem 4 US

You can make a right triangle ACG which you can use to calculate AG.

US Webpages_ Pythagorean Theorem 5 US

In order to use Pythagoras’ Theorem, you need to know two sides of the triangle. So in order to figure out the longest side AG, you first need to figure out one of the shorter sides AC.

Let’s call this side x and redraw this triangle.

US Webpages_ Pythagorean Theorem 6 US

You can see that the side labeled x forms the diagonal line of the base of the rectangular prism.

US Webpages_ Pythagorean Theorem 7 US

Triangle ABC is a right triangle, so we can use the Pythagorean Theorem to calculate x.

x=\sqrt{10^2 + 4^2} = 2\sqrt{29} = 10.7703…

AG=\sqrt{10.7703…^2 + 6^2} = \sqrt{152}=2\sqrt{38} = 12.328…

So the required length is 12.4 \, cm (rounded to the nearest tenth).

Step-by-step guide: 3D Pythagorean Theorem

What is the Pythagorean Theorem?

Common Core State Standards

How does this relate to 8 th grade math?

  • Grade 8 – Geometry (8.G.B.6) Explain a proof of the Pythagorean Theorem and its converse.
  • Grade 8 – Geometry (8.G.B.7) Apply the Pythagorean Theorem to determine unknown side lengths in right triangles in real-world and mathematical problems in two and three dimensions.

How to use Pythagorean Theorem

In order to use Pythagorean Theorem:

Label the sides of the triangle.

Write down the formula and apply the numbers.

Record the answer.

Pythagorean Theorem examples

Example 1: missing length of the hypotenuse c.

Find x and answer to the nearest hundredth.

US Webpages_ Pythagorean Theorem 8 US

Label the hypotenuse (the longest side) with c. The adjacent sides, next to the right angle can be labeled a and b (either way – they are interchangeable).

US Webpages_ Pythagorean Theorem 9 US

2 Write down the formula and apply the numbers.

\begin{aligned}& a^2+b^2=c^2 \\\\ & 3^2+8^2=x^2 \\\\ & 9+64=x^2 \\\\ & 73=x^2 \\\\ & \sqrt{73}=x\end{aligned}

An alternative method of rearranging the formula and to put one calculation into a calculator will also work.

\begin{aligned}& a^2+b^2=c^2 \\\\ & c^2=a^2+b^2 \\\\ & c=\sqrt{a^2+b^2} \\\\ & x=\sqrt{3^2+8^2}\end{aligned}

3 Record the answer.

Make sure you give your final answer in the correct form by calculating the square root value, including units where appropriate.

x=\sqrt{73}=8.5440037…

The final answer to the nearest hundredth is:

x=8.54 \mathrm{~cm}

Example 2: missing length c

Find x and answer to the nearest tenth.

US Webpages_ Pythagorean Theorem 10 US

\begin{aligned}& a^2 + b^2 = c^2 \\\\ & 7^2 + 9^2 = x^2 \\\\ & x^2 = 7^2 + 9^2 \\\\ & x^2 = 49+81 \\\\ & x^2 = 130 \\\\ & x = \sqrt{130}\end{aligned}

An alternative method is to rearrange the formula and put one calculation into a calculator.

\begin{aligned}& a^2+b^2=c^2 \\\\ & c^2=a^2+b^2 \\\\ & c=\sqrt{a^2+b^2} \\\\ & x=\sqrt{7^2+9^2}\end{aligned}

Make sure you give your final answer in the correct form; including units where appropriate.

x=\sqrt{130}=11.40175.…

The final answer to the nearest tenth is:

x=11.4 \mathrm{~cm}

Example 3: finding an adjacent side (a short side)

Find x and write your answer to the nearest hundredth.

US Webpages_ Pythagorean Theorem 12 US

\begin{aligned} & a^2 + b^2 = c^2 \\\\ & x^2 + 5^2 = 8^2 \\\\ & x^2+25 = 64 \\\\ & x^2 = 64 - 25 \\\\ & x^2 = 39 \\\\ & x =\sqrt{39}\ \end{aligned}

\begin{aligned}& a^2+b^2=c^2 \\\\ & a^2=c^2-b^2 \\\\ & a=\sqrt{c^2-b^2} \\\\ & x=\sqrt{8^2-5^2}\end{aligned}

Make sure you give your final answer in the correct form, including units where appropriate.

x=\sqrt{39}=6.244997.…

x=6.24 \mathrm{~cm}

Example 4: finding an adjacent side (a short side)

Find x and write your answer to the nearest tenth.

US Webpages_ Pythagorean Theorem 14 US

\begin{aligned} & a^2 + b^2 = c^2 \\\\ & x^2 + 11^2 = 20^2\\\\ & x^2+121 = 400 \\\\ & x^2= 400 - 121 \\\\ & x^2 = 279\\\\ & x =\sqrt{279}\end{aligned}

\begin{aligned}& a^2+b^2=c^2 \\\\ & a^2=c^2-b^2 \\\\ & a=\sqrt{c^2-b^2} \\\\ & x=\sqrt{20^2-11^2}\end{aligned}

x=\sqrt{279}=16.70329.…

The final answer rounded to the nearest tenth is:

x=16.7 \mathrm{~cm}

Example 5: checking if a triangle has a right angle

Is the triangle below a right triangle?

US Webpages_ Pythagorean Theorem 16 US

Label the longest side with c. The adjacent sides, next to the right angle can be labeled a and b (either way – they are interchangeable).

US Webpages_ Pythagorean Theorem 17 US

\begin{aligned} & a^2 + b^2 = c^2 \\\\ & 8^2 + 10^2 = 13^2 \\\\ & 64+100 = 169 \\\\ & 164 = 169\end{aligned}

But this is NOT correct. Pythagorean Theorem only works with right triangles.

Because 8^2 + 10^2 ≠ 13^2 , the sides of the triangles do not fit with Pythagorean Theorem. Therefore, the triangle is NOT a right triangle and c is not a hypotenuse.

Example 6: checking if a triangle has a right angle

US Webpages_ Pythagorean Theorem 18 US

\begin{aligned}& a^2 + b^2 = c^2 \\\\ & 6^2 + 8^2 = 10^2 \\\\ & 36+64 = 100 \\\\ & 100 = 100\end{aligned}

This is correct. Pythagorean Theorem only works with right triangles.

Because 6^2 + 8^2 = 10^2 , the sides of the triangles fit with Pythagorean Theorem. Therefore, the triangle is a right triangle and c is a hypotenuse.

Teaching tips for Pythagorean Theorem

  • In the beginning, give examples that are on grids and have the sides of the right triangle as positive integers. This allows students to draw the corresponding square for each side of the triangle and test to see that a^2+b^2=c^2.
  • Give students a chance to try and prove the Pythagorean Theorem on their own. Then give students examples of a few different theorems and challenge them to find the one that makes the most sense for them.
  • The Pythagorean Theorem has connections outside of math classes. If time allows, students can explore the history of this theorem.

Easy mistakes to make

  • Not correctly identifying the hypotenuse It is very important to make sure that the hypotenuse, the long side, is correctly identified and labeled c.

US Webpages_ Pythagorean Theorem 20 US

  • Thinking the lengths of sides can only be whole numbers Lengths can be decimals, fractions or even irrational numbers such as, \sqrt{2}.

US Webpages_ Pythagorean Theorem 21 US

  • Rounding too early If you need to use Pythagorean Theorem in a question with multiple steps, do not round until the very end of the question or you will lose accuracy. For example, you may need to find the height of a triangle, and then use that height to find its area.

Practice Pythagorean Theorem questions

1. Find side x. Give your answer to the nearest hundredth:

US Webpages_ Pythagorean Theorem 22 US

\begin{aligned}& a^2 + b^2 = c^2 \\\\ & 7^2 + 5^2 = x^2 \\\\ & x^2 = 7^2 + 5^2\\\\ & x^2 = 49+25 \\\\ & x^2 = 74 \\\\& x = \sqrt{74}\\\\ &x = 8.602325…\end{aligned}

x=8.60 \mathrm{~cm}

2. Find side x. Give your answer to the nearest hundredth:

pythagorean theorem problem solving questions

\begin{aligned}&a^2 + b^2 = c^2 \\\\ & 14^2 + 10^2 = x^2 \\\\ & x^2 = 14^2 + 10^2 \\\\ & x^2 = 196+10 \\\\ & x^2 = 296 \\\\ & x = \sqrt{296} \\\\ & x = 17.20465… \end{aligned}

x=17.20 \mathrm{~cm}

3. Find side x. Give your answer to the nearest hundredth:

US Webpages_ Pythagorean Theorem 26 US

\begin{aligned}& a^2 + b^2 = c^2 \\\\ & x^2 + 10^2 = 18^2\\\\ & x^2+100 = 324 \\\\ & x^2 = 324 – 100 \\\\ & x^2 = 224 \\\\ & x =\sqrt{224} \\\\ & x = 14.96662…\end{aligned}

x=14.97 \mathrm{~cm}

4. Find side x. Give your answer to the nearest hundredth:

US Webpages_ Pythagorean Theorem 28 US

\begin{aligned}& a^2 + b^2 = c^2 \\\\ & x^2 + 2.5^2 = 7.2^2 \\\\ & x^2+6.25 = 51.84 \\\\ & x^2 = 51.84-6.25 \\\\ & x^2 = 45.59 \\\\ & x =\sqrt{45.59} \\\\ & x = 6.75203…\end{aligned}

x=6.75 \mathrm{~cm}

5. Is this a right triangle? Justify your answer with the Pythagorean Theorem.

US Webpages_ Pythagorean Theorem 30 US

No, because 12^2+5^2 ≠ 13^2

Yes, because I measured the angle and it was 90^{\circ}

Yes, because 12^2+5^2=13^2

No, because I measured the angle and it was not 90^{\circ}

US Webpages_ Pythagorean Theorem 31 US

\begin{aligned}& a^2 + b^2 = c^2 \\\\ & 12^2 + 5^2 = 13^2 \\\\ & 144+25 = 169 \\\\ & 169 = 169 \end{aligned}

Therefore the triangle is a right triangle.

6. Is this a right triangle? Justify your answer with the Pythagorean Theorem.

US Webpages_ Pythagorean Theorem 32 US

Yes, because 6^2+13^2=14^2

No, because 6^2+13^2 ≠ 14^2

US Webpages_ Pythagorean Theorem 33 US

\begin{aligned}& a^2 + b^2 = c^2 \\\\ & 6^2 + 13^2 = 14^2 \\\\ & 36+169 = 196 \\\\ & 205 = 196\end{aligned}

This is NOT correct. Pythagorean Theorem only works with right triangles.

Therefore the triangle is NOT a right-angled triangle.

Pythagorean Theorem FAQs

Pythagorean Theorem is named after a Greek mathematician who lived about 2,500 years ago, however, the ancient Babylonians used this rule about 4 thousand years ago! At the same time, the Egyptians were using the theorem to help them with right angles when building structures.

No, this is only true for the sides of a right triangle. The sum of the squares of the lengths for a and b will only be equal to the square of side c if the triangle is right. However, you can use this relationship to decide if a triangle is acute or obtuse. For an acute triangle, the square of the hypotenuse will be less than the sum of the squares of a and b. For an obtuse triangle, the square of the hypotenuse will be more than the sum of the squares of a and b.

Yes, there are many algebraic proofs and geometric proofs that address the Pythagorean Theorem. The proof shown at the top of this page is one of the simplest ways to prove the Pythagorean Theorem.

No, though they all follow the Pythagorean Theorem this does not mean they are similar. They are only similar if there is a multiplicative relationship between each corresponding side of the triangle.

The next lessons are

  • Trigonometry
  • Circle math
  • Sectors, arcs and segments

Still stuck?

At Third Space Learning, we specialize in helping teachers and school leaders to provide personalized math support for more of their students through high-quality, online one-on-one math tutoring delivered by subject experts.

Each week, our tutors support thousands of students who are at risk of not meeting their grade-level expectations, and help accelerate their progress and boost their confidence.

One on one math tuition

Find out how we can help your students achieve success with our math tutoring programs .

[FREE] Common Core Practice Tests (3rd to 8th Grade)

Prepare for math tests in your state with these 3rd Grade to 8th Grade practice assessments for Common Core and state equivalents.

Get your 6 multiple choice practice tests with detailed answers to support test prep, created by US math teachers for US math teachers!

Privacy Overview

  • High School
  • High school geometry
  • Pythagorean Theorem

pythagorean theorem problem solving questions

Expert Guidance for Your Pythagorean Theorem Questions

Hayley Steele

Two congruent circles with centres at (2,3) and (5,6), which intersect at right angles, have radius equal to?

Theresa Daugherty

State whether the given statement is true or false: 9, 40, 41 is a Pythagorean triplet. True or false

nefg4m

5, 12, 13 is a Pythagorean triplet. True or false

glucidessho

8, 15, 17 is a Pythagorean triplet

hEorpaigh3tR

Based on Pythagorean identities, which equation is true? A. sin 2 ⁡ θ − 1 = cos 2 ⁡ θ B. sec 2 ⁡ θ − tan 2 ⁡ θ = − 1 C. - cos 2 ⁡ θ − 1 = sin 2 ⁡ θ D. cot 2 ⁡ θ − csc 2 ⁡ θ = − 1

Scarlet Marshall

The end rollers of bar AB(1.5R) are constrained to the slot. If roller A has a downward velocity of 1.2 m/s and this speed is constant over a small motion interval, determine the tangential acceleration of roller B as it passes the topmost position. The value of R is 0.5 m.

umthumaL3e

The area of the obtuse angle triangle shown below is: A17.5 sq. units B 25 sq. units C 14 sq. units D 15 sq. units

autreimL8

Replacing x with 1 2 in 2 x 2 will give you an answer of 1.

Neil Sharp

Why does the Pythagorean theorem only work for right triangles?

BertonCO5

7, 24, 25 is a Pythagorean triplet. A.True B.False

Widersinnby7

A bicycle wheel with a 5 inch ray rotates 60 ∘ . What distance has the bicycle traveled?

calcific5z

The legs of a right triangle are 6 and 8 cm. Find the hypotenuse and the area of ​​the triangle.

Karli Kidd

Solve for X. Anlge A=8x+5 Angle B=4 x 2 -10 Angle C= x 2 +2x+10 I know that they equal 180 degrees. However I am drawinga blank on the factoring part of it

obojeneqk

For each of the following, can the measures represent sides ofa right triangle? Explain your answers.a. 3 m, 4 m, 5 mb. 2 c m , 3 c m , 5 c m

on2t1inf8b

A 10-m ladder is leaning against a building. The bottom of theladder is 5-m from the building. How high is the top of theladder?

Ryan Robertson

Pythagorean theorem and its cause I'm in high school, and one of my problems with geometry is the Pythagorean theorem. I'm very curious, and everything I learn, I ask "but why?". I've reached a point where I understand what the Pythagorean theorem is, and I understand the equation, but I can't understand why it is that way. Like many things in math, I came to the conclusion that it is that way because it is; math is the laws of the universe, and it may reach a point where the "why" answers itself. So what I want to know is, is there an explication to why the addition of the squared lengths of the smaller sides is equal to the squared hypotenuse, or is it just a characteristic of the right triangle itself? And is math the answer to itself? Thank you.

kramberol

pythagorean theorem extensions are there for a given integer N solutions to the equations ∑ n = 1 N x i 2 = z 2 for integers x i and zan easier equation given an integer number 'a' can be there solutions to the equation ∑ n = 1 N x i 2 = a 2 for N=2 this is pythagorean theorem

sweetymoeyz

"Pythagorean theorem" for projection onto convex set I'm going through the book on online convex optimization by Hazan, and in the first chapter I saw this assertion (which Hazan calls the "pythagorean theorem"): Let K ⊂ R d be a convex set, y ∈ R d , and x = Π K ( y ) . Then for any z ∈ K we have: ‖ y − z ‖ ≥ ‖ x − z ‖ . It is presented without proof - what is a proof for this? Also, how does it relate to the pythagorean theorem?

Janessa Olson

Non-geometric Proof of Pythagorean Theorem Is there a purely algebraic proof for the Pythagorean theorem that doesn't rely on a geometric representation? Just algebra/calculus. I want to TRULY understand the WHY of how it is true. I know it works and I know the geometric proofs.

2d3vljtq

The Pythagorean theorem and Hilbert axioms Can one state and prove the Pythagorean theorem using Hilbert's axioms of geometry, without any reference to arithmetic? Edit: Here is a possible motivation for this question (and in particular for the "state" part of this question). It is known that the theory of Euclidean geometry is complete. Every true statement in this theory is provable. On the other hand, it is known that the axioms of (Peano) arithmetic cannot be proven to be consistent. So, basically, I ask if there is a reasonable theory which is known to be consistent and complete, and in which the Pythagorean theorem can be stated and proved. In summary, I guess I am asking - can we be sure that the Pythagorean theorem is true? :)

next

  • Solid Geometry
  • Conic sections
  • Analytic geometry
  • Transformation properties
  • Performing transformations
  • Maximization
  • Arc of a Circle
  • Geometric Probability
  • Angle theorems
  • Finding volume
  • Distance Formula
  • Indirect Proof
  • Vertices Of An Ellipse
  • Quadrilaterals
  • Logical Reasoning
  • High School Questions
  • College Questions
  • Math Solver
  • Top Questions
  • Top Questions 2
  • Term of Service
  • Payment Policy

Connect with us

Get Plainmath App

  • Google Play

Plainmath Logo

E-mail us: [email protected]

Our Service is useful for:

Plainmath is a platform aimed to help users to understand how to solve math problems by providing accumulated knowledge on different topics and accessible examples.

2023 Plainmath. All rights reserved

Corbettmaths

Pythagoras Practice Questions

Click here for questions, click here for answers.

GCSE Revision Cards

pythagorean theorem problem solving questions

5-a-day Workbooks

pythagorean theorem problem solving questions

Primary Study Cards

pythagorean theorem problem solving questions

Privacy Policy

Terms and Conditions

Corbettmaths © 2012 – 2024

Pythagoras Theorem Questions (with Answers)

Pythagoras theorem.

In a right triangle, the square of the hypotenuse is equal to the sum of squares of the other two sides.

Right Triangle with Pythagoras Statement

  • Length of the hypotenuse is c
  • The hypotenuse is the longest side
  • Lengths of the other sides are a, b

Right Triangle Questions – using the theorem

The Theorem helps us in:

  • Finding Sides: If two sides are known, we can find the third side.
  • Determining if a triangle is right-angled: If the sides of a triangle are known and satisfy the Pythagoras Formula, it is a right-angled triangle.

There is a proof of this theorem by a US president. Its simplicity makes it is easy enough for the grade 8 kids to understand.

Finding the missing sides (side lengths) of a Right Triangle

The theorem gives a relation among the three sides of a right-angled triangle. We can find one side if we know the other two sides. How?

Example: We are given (see figure) below the two sides of the right triangle. Find the third side.

Right triangle with unknown side

Given: a = 3, c = 5

Which side is the hypotenuse?

✩ Always identify the hypotenuse first

Unknown side = BC = b ?

Putting the values in the Pythagoras Formula: a 2 + b 2 = c 2

3 2 + b 2 = 5 2

9 + b 2 = 25

b 2 = 25 − 9 = 16 = 4 2

Finding the Hypotenuse of a Triangle

Using the Pythagoras formula, finding hypotenuse is no different from any other side.

Example: Sides of a right triangle are 20 cm and 21 cm, find its hypotenuse.

Right triangle with unknown hypotenuse

Pythagoras Formula: a 2 + b 2 = c 2

  • c is the length of the hypotenuse
  • a, b are the lengths of the other two sides (you can assume any length as a or b ).

Let AB = a = 20, BC = b = 21

Putting values in the formula:

20 2 + 21 2 = c 2

400 + 441 = c 2

Finding Right Triangle

Given the sides, we can determine if a triangle is right-angled by applying the Pythagoras Formula. How?

  • Assume the longest side to be hypotenuse Length = c . Find its square ( = c 2 )
  • Find the sum of squares of the other two sides ( = a 2 + b 2 )
  • If a 2 + b 2 ≠ c 2 it is a not right triangle
  • If a 2 + b 2 = c 2 it is a right triangle

Example: A triangle has sides 8 cm, 11 cm, and 15 cm. Determine if it is a right triangle

Longest side = 15 cm. Let us assume it to be hypotenuse = c (as we know that it is always the longest)

c 2 = 15 2 = 225

Other sides a = 8 cm b = 11 cm. (You can assume any side length to be a or b ).

a 2 + b 2 = 8 2 + 11 2 = 64 + 121 = 185

a 2 + b 2 ≠ c 2

So this is not a right-angled triangle.

Example: The sides of a triangle are 8 cm, 17 cm, and 15 cm. Find if it is a right triangle.

Longest side = 17 cm. Let us assume it to be hypotenuse = c

c 2 = 17 2 = 289

Other sides a = 8 cm, b = 15 cm.

a 2 + b 2 = 8 2 + 15 2 = 64 + 225 = 289

So a 2 + b 2 = c 2

It is a right-angled triangle.

Pythagoras Questions Types

You will encounter the following types of questions related to this theorem:

  • Find a side, given two sides These questions are the direct application of the theorem (formula) and are easiest to solve.
  • Express the relation between the two sides in an equation
  • Substitute one side by the other using the first equation in the Pythagoras Formula
  • Given the Perimeter and one side, find other sides – Perimeter is the sum of the three sides. Since one side is known, we subtract it from the perimeter to get a relationship between the other two sides.
  • Given Area and one side find other sides – Area = 2 1 ​ × ( ba se × a lt i t u d e ) . Base and altitude can be the sides with the right angle OR the hypotenuse and the altitude.

Pythagoras Questions

The questions chosen have minimal use of other concepts, yet, some of these are hard Pythagoras questions (See Ques 4 and Ques 10 ).

1 Question

ABC is a right triangle. AC is its hypotenuse. Length of side AB is 2√5 . Side BC is twice of side AB. Find the length of AC.

Can you express BC in terms of AB and apply the Pythagoras Theorem?

Ans.   AC = 10

2 Question

The hypotenuse of a right triangle is 6 cm . Its area is 9 cm 2 . Find its sides.

Right triangle ABC with hypotenuse AC = 6 cm

Can you form two equations – one using area and the other using the Pythagoras formula?

Ans.   Each side is 3√2 cm.

3 Question

One side of a right triangle is 4 10 ​ cm. Find the length of its other side if the hypotenuse is 13 cm.

Can you directly apply the Pythagoras Theorem?

Ans.   3 cm

4 Question

In a right triangle ABC, length of the medians to the sides AB and BC are 2 61 ​ and 601 ​ respectively. Find the length of its hypotenuse.

Right triangle ABC with medians to side AB and BC

CE = 2 61 ​

Let AE = EB = x

BD = DC = y

Can you use the Pythagorean Theorem to form an equation between x, y, and AD?

Can you form another equation involving x, y, and EC?

Can you use this result to find A C 2 ? Use the triangle ABC.

Ans.   AC = 26

5 Question

In a right triangle, the longest side is 8 cm. One of the remaining sides is 4√3 cm long. Find the length of the other side.

Can you apply the Pythagoras Theorem directly?

Ans.   4 cm

6 Question

The first side of a right triangle is shorter than the second side by 1 cm. It is longer than the third side by 31 cm. Find the sides of the triangle.

Can you form equations between the first side and the other two sides? Which side is the hypotenuse?

Ans.   9 cm, 40 cm, 41 cm

7 Question

The perimeter of a right triangle is equal to 30 cm. The length of one of its sides is 10 cm. Find its hypotenuse.

Right triangle ABC with hypotenuse AC

Can you find the relation between the unknown side and the perimeter in terms of the hypotenuse?

Ans.   12.5 cm

8 Question

The sides of a triangle are 5 cm, 9 cm, and 12 cm. Is it a right-angled triangle?

Can you identify the possible hypotenuse? Also, test if the sides satisfy the Pythagoras Formula.

Ans.   Not a right triangle.

9 Question

In a right triangle, two sides are equal. The longest side is 7√2 cm, find the remaining sides.

Ans.   7 cm

10 Question

In the following right triangle altitude BD = 9 10 ​ cm and DC = 27 10 ​ cm. Find the sides of the triangle.

Right triangle ABC with altitude BD to the hypotenuse AC

Can you apply the Pythagoras Theorem to the triangle BCD?

Can you form an equation between a and x using the triangle ABD?

Can you find the value of x using above equations and triangle ABC?

Ans.   AB = 30cm, BC = 90cm, A C = 30 10 ​ c m

Answers to Pythagoras Questions

1 answer.

Let AB = a , BC = b and AC = c .

AB = a = 2√5

BC is twice of AB, b = 2a = 4√5

AC = Hypotenuse = c

Applying the Pythagoras Theorem a 2 + b 2 = c 2 :

(2√5) 2 + (4√5) 2 = c 2

4(5) + 16(5) = c 2

c 2 = 20 + 80 = 100

2 Answer

Let AB = a , BC = b

In triangle ABC, base = b and altitude = a

Area of Triangle = 2 1 ​ × ( ba se × a lt i t u d e ) . So:

2 1 ​ × ( ab ) = 9

ab = 18 (Equation 1)

Using the Pythagoras Formula:

a 2 + b 2 = 6 2 = 36

We add and subtract 2ab to complete the square :

a 2 + b 2 − 2ab + 2ab = 36

(a − b) 2 + 2ab = 36

(a − b) 2 + 36 = 36 Using ab = 18 from Equation 1

(a − b) 2 = 0

Substituting a by b in Equation 1:

Side AB = BC = 3√2 cm

3 Answer

Let the length of sides be a, b and c , such that:

a = 4 10 ​ cm

b = unknown

From the Pythagoras formula a 2 + b 2 = c 2 , we get:

( 4 10 ​ ) 2 + b 2 = 1 3 2

(16 × 10) + b 2 = 169

160 + b 2 = 169

b 2 = 169 − 160 = 9

4 Answer

Using triangle ABD:

AB 2 + BD 2 = AD 2

( 2 x ) 2 + y 2 = ( 601 ​ ) 2

4 x 2 + y 2 = 601 (Equation 1)

Using triangle EBC :

EB 2 + BC 2 = EC 2

x 2 + ( 2 y ) 2 = ( 2 61 ​ ) 2

x 2 + 4 y 2 = ( 2 61 ​ ) 2

x 2 + 4 y 2 = 244 (Equation 2)

Adding Equation 1 and 2:

4x 2 + y 2 + x 2 + 4y 2 = 601 + 244

5x 2 + 5y 2 = 845

x 2 + y 2 = 169 (Equation 3)

Let us solve for the hypotenuse using the triangle ABC:

AB 2 + BC 2 = AC 2

(2x) 2 + (2y) 2 = AC 2

4x 2 + 4y 2 = AC 2

4(x 2 + y 2 ) = AC 2

Substituting the value of x 2 + y 2 from Equation 3:

4(169) = AC 2

A C = 4 ( 169 ) ​

AC = 2 × 13 = 26

5 Answer

Let the lengths of sides be a, b and c (hypotenuse).

Hypotenuse is the longest side. So c = 8 .

Let b = 4√3 .

From the Pythagoras Theorem:

a 2 + b 2 = c 2

a 2 + (4√3) 2 = 8 2

a 2 + 16(3) = 64

a 2 + 48 = 64

The third side is 4 cm.

6 Answer

The second side is the longest. It is the hypotenuse. Let its length be c .

Let the length of first side be b and third side a .

Applying the Pythagoras formula:

(b − 31) 2 + b 2 = (b + 1) 2

b 2 − 62b + 31 2 + b 2 = b 2 + 2b + 1

b 2 − 62b + 961 = 2b + 1

b 2 − 64b + 960 = 0

b 2 − 24b − 40b + 960 = 0

b(b − 24) − 40(b − 24) = 0

(b − 40)(b − 24) = 0

b = 40 Or b = 24

For b = 24 , we get a = 24 − 31 = − 7 . Length of a side cannot be negative, so we reject b = 24 .

For b = 40 , we get a = 40 − 31 = 9 and c = 40 + 1 = 41

The sides of triangle are 9 cm, 40 cm and 41 cm.

7 Answer

Side BC = b = 10 cm

Perimeter = Sum of the sides

= a + b + c = 30 (Given)

a + 10 + c = 30

a = 20 − c ( Equation 1 )

Applying the Pythagoras Theorem to find the hypotenuse:

Using Equation 1 to substitute the value of a

(20 − c) 2 + (10) 2 = c 2

400 − 40c + c 2 + 100 = c 2

500 − 40c = 0

The length of hypotenuse = 12.5 cm

8 Answer

Longest side = 12 cm. Let us assume it to be the hypotenuse = c

So c 2 = 12 2 = 144

The Pythagoras Formula: a 2 + b 2 = c 2

We can assume any side to be a or b.

Let a = 5 cm, b = 9 cm.

a 2 + b 2 = 5 2 + 9 2 = 25 + 81 = 106

So a 2 + b 2 ≠ c 2

This is a not a right angled triangle.

9 Answer

Let the length of the sides be a, b , and c (hypotenuse).

In a right triangle hypotenuse is the longest side. So c = 7√2

Other sides are equal. So a = b .

Applying the Pythagoras theorem:

b 2 + b 2 = (7√2) 2

2b 2 = 49(2)

Each side is 7 cm.

10 Answer

Let AB = a, BC = b, AC = c and AD = x

Given B D = 9 10 ​ D C = 27 10 ​

Applying Pythagoras Theorem to triangle BCD:

BD 2 + DC 2 = BC 2

( 9 10 ​ ) 2 + ( 27 10 ​ ) 2 = B C 2

(9 2 × 10) + (27 2 × 10) = b 2

810 + 7290 = b 2

Applying Pythagoras Theorem to triangle ABD:

BD 2 + AD 2 = AB 2

a 2 = ( 9 10 ​ ) 2 + x 2 (Equation 1)

From the figure:

AC = AD + BD

c = x + 27 10 ​ (Equation 2)

Applying Pythagoras Theorem to triangle ABC:

Using b = 90 and value of a 2 from Equation 1 and c from Equation 2:

( 9 10 ​ ) 2 + x 2 + 9 0 2 = ( x + 27 10 ​ ) 2

9 2 ( 10 ) + x 2 + 9 0 2 = x 2 + 54 10 ​ x + 2 7 2 ( 10 )

810 + 8100 = 54 10 ​ x + 7290

54 10 ​ x = 8910 − 7290 = 1620

x = 10 ​ 30 ​

Putting value of x in Equation 1:

a 2 = ( 9 10 ​ ) 2 + ( 3 10 ​ ) 2

a 2 = 810 + 90 = 900

Using value of a and b in Pythagoras Formula for triangle ABC:

30 2 + 90 2 = c 2

900 + 8100 = c 2

c = 30 10 ​

AB = 30cm, BC = 90cm, A C = 30 10 ​ c m

Difficult Pythagoras Questions (Year 10, Guided Answers) ➤

James Garfield Pythagorean Theorem (Illustration & Proof) ➤

Math Problems, Tests, Forums

  • Math Forum/Help
  • Problem Solver
  • College Math
  • Pythagorean Theorem

Pythagorean Theorem: Problems with Solutions

pythagorean theorem problem solving questions

Word problems on Pythagorean Theorem

Learn how to solve different types of word problems on Pythagorean Theorem .

Pythagoras Theorem can be used to solve the problems step-by-step when we know the length of two sides of a right angled triangle and we need to get the length of the third side.

Three cases of word problems on Pythagorean Theorem :

Case 1: To find the hypotenuse where perpendicular and base are given.

Case 2: To find the base where perpendicular and hypotenuse are given.

Case 3: To find the perpendicular where base and hypotenuse are given.

Word problems using the Pythagorean Theorem:

1. A person has to walk 100 m to go from position X in the north of east direction to the position B and then to the west of Y to reach finally at position Z. The position Z is situated at the north of X and at a distance of 60 m from X. Find the distance between X and Y.

Let XY = x m

Therefore, YZ = (100 – x) m

In ∆ XYZ, ∠Z = 90°


Therefore, by Pythagoras theorem

XY = YZ + XZ

⇒ x = (100 – x) + 60

⇒ = 10000 – 200x + + 3600

Pythagorean Theorem Word Problem

⇒ 200x = 10000 + 3600

⇒ 200x = 13600

⇒ x = 13600/200

Therefore, distance between X and Y = 68 meters.

Word problems on Pythagorean Theorem

Therefore, length of each side is 8 cm.

Using the formula solve more word problems on Pythagorean Theorem.

3. Find the perimeter of a rectangle whose length is 150 m and the diagonal is 170 m.

Word problem on Pythagorean Theorem

In a rectangle, each angle measures 90°.

Therefore PSR is right angled at S

Using Pythagoras theorem, we get

⇒ PS = √6400

Therefore perimeter of the rectangle PQRS = 2 (length + width)

                                                          = 2 (150 + 80) m

                                                          = 2 (230) m

                                                          = 460 m

4. A ladder 13 m long is placed on the ground in such a way that it touches the top of a vertical wall 12 m high. Find the distance of the foot of the ladder from the bottom of the wall.

Word problems using the Pythagorean Theorem

Let the required distance be x meters. Here, the ladder, the wall and the ground from a right-angled triangle. The ladder is the hypotenuse of that triangle.

According to Pythagorean Theorem,

Therefore, distance of the foot of the ladder from the bottom of the wall = 5 meters.

5. The height of two building is 34 m and 29 m respectively. If the distance between the two building is 12 m, find the distance between their tops.

Pythagorean Theorem: Word Problems

The vertical buildings AB and CD are 34 m and 29 m respectively.

Draw DE ┴ AB

Then AE = AB – EB but EB = BC

Therefore AE = 34 m - 29 m = 5 m

Now, AED is right angled triangle and right angled at E.

⇒ AD = √169

Therefore the distance between their tops = 13 m.

The examples will help us to solve various types of word problems on Pythagorean Theorem.

Congruent Shapes

Congruent Line-segments

Congruent Angles

Congruent Triangles

Conditions for the Congruence of Triangles

Side Side Side Congruence

Side Angle Side Congruence

Angle Side Angle Congruence

Angle Angle Side Congruence

Right Angle Hypotenuse Side congruence

Pythagorean Theorem

Proof of Pythagorean Theorem

Converse of Pythagorean Theorem

7th Grade Math Problems 8th Grade Math Practice From Word problems on Pythagorean Theorem to HOME PAGE

Didn't find what you were looking for? Or want to know more information about Math Only Math . Use this Google Search to find what you need.

New! Comments

What’s this?

Facebook X Pinterest WhatsApp Messenger
  • Preschool Activities
  • Kindergarten Math
  • 1st Grade Math
  • 2nd Grade Math
  • 3rd Grade Math
  • 4th Grade Math
  • 5th Grade Math
  • 6th Grade Math
  • 7th Grade Math
  • 8th Grade Math
  • 9th Grade Math
  • 10th Grade Math
  • 11 & 12 Grade Math
  • Concepts of Sets
  • Probability
  • Boolean Algebra
  • Math Coloring Pages
  • Multiplication Table
  • Cool Maths Games
  • Math Flash Cards
  • Online Math Quiz
  • Math Puzzles
  • Binary System
  • Math Dictionary
  • Conversion Chart
  • Homework Sheets
  • Math Problem Ans
  • Free Math Answers
  • Printable Math Sheet
  • Funny Math Answers
  • Employment Test
  • Math Patterns
  • Link Partners
  • Privacy Policy
E-mail Address
First Name

to send you Math Only Math.

Recent Articles

Worksheet on unitary method | unitary method word problems | answers.

Aug 02, 24 04:40 PM

Worksheet on Unitary Method

Unitary Method | Learn the Basics of Unitary Method | Unitary Formula

Aug 02, 24 04:13 PM

Unitary Method

Divide a Number into Three Parts in a Given Ratio | Solved Examples

Aug 01, 24 02:50 PM

Worksheet on Proportions | Word Problems | Continued & Mean Proportion

Aug 01, 24 02:47 PM

Concept of Proportion | Definitions | Solved Examples | Free Worksheet

Aug 01, 24 12:50 PM

© and ™ math-only-math.com. All Rights Reserved. 2010 - 2024.

MathBootCamps

The pythagorean theorem with examples.

The Pythagorean theorem is a way of relating the leg lengths of a right triangle to the length of the hypotenuse, which is the side opposite the right angle. Even though it is written in these terms, it can be used to find any of the side as long as you know the lengths of the other two sides. In this lesson, we will look at several different types of examples of applying this theorem.

Table of Contents

  • Examples of using the Pythagorean theorem
  • Solving applied problems (word problems)
  • Solving algebraic problems

[adsenseWide]

Applying the Pythagorean theorem (examples)

In the examples below, we will see how to apply this rule to find any side of a right triangle triangle. As in the formula below, we will let a and b be the lengths of the legs and c be the length of the hypotenuse. Remember though, that you could use any variables to represent these lengths.

image showing the pythagorean theorem formula

In each example, pay close attention to the information given and what we are trying to find. This helps you determine the correct values to use in the different parts of the formula.

Find the value of \(x\).

triangle with unknown hypotenuse

The side opposite the right angle is the side labelled \(x\). This is the hypotenuse. When applying the Pythagorean theorem, this squared is equal to the sum of the other two sides squared. Mathematically, this means:

\(6^2 + 8^2 = x^2\)

Which is the same as:

\(100 = x^2\)

Therefore, we can write:

\(\begin{align}x &= \sqrt{100}\\ &= \bbox[border: 1px solid black; padding: 2px]{10}\end{align}\)

Maybe you remember that in an equation like this, \(x\) could also be –10, since –10 squared is also 100. But, the length of any side of a triangle can never be negative and therefore we only consider the positive square root.

In other situations, you will be trying to find the length of one of the legs of a right triangle. You can still use the Pythagorean theorem in these types of problems, but you will need to be careful about the order you use the values in the formula.

Find the value of \(y\).

example where a leg length is unknown

The side opposite the right angle has a length of 12. Therefore, we will write:

\(8^2 + y^2 = 12^2\)

This is the same as:

\(64 + y^2 = 144\)

Subtracting 64 from both sides:

\(y^2 = 80\)

\(\begin{align}y &= \sqrt{80} \\ &= \sqrt{16 \times 5} \\ &= \bbox[border: 1px solid black; padding: 2px]{4\sqrt{5}}\end{align}\)

In this last example, we left the answer in exact form instead of finding a decimal approximation. This is common unless you are working on an applied problem.

Applications (word problems) with the Pythagorean theorem

There are many different kinds of real-life problems that can be solved using the Pythagorean theorem. The easiest way to see that you should be applying this theorem is by drawing a picture of whatever situation is described.

Two hikers leave a cabin at the same time, one heading due south and the other headed due west. After one hour, the hiker walking south has covered 2.8 miles and the hiker walking west has covered 3.1 miles. At that moment, what is the shortest distance between the two hikers?

First, sketch a picture of the information given. Label any unknown value with a variable name, like x.

figure for a word problem with the pythagorean theorem

Due south and due west form a right angle, and the shortest distance between any two points is a straight line. Therefore, we can apply the Pythagorean theorem and write:

\(3.1^2 + 2.8^2 = x^2\)

Here, you will need to use a calculator to simplify the left-hand side:

\(17.45 = x^2\)

Now use your calculator to take the square root. You will likely need to round your answer.

\(\begin{align}x &= \sqrt{17.45} \\ &\approx 4.18 \text{ miles}\end{align}\)

As you can see, it will be up to you to determine that a right angle is part of the situation given in the word problem. If it isn’t, then you can’t use the Pythagorean theorem.

Algebra style problems with the Pythagorean theorem

There is one last type of problem you might run into where you use the Pythagorean theorem to write some type of algebraic expression. This is something that you will not need to do in every course, but it does come up.

A right triangle has a hypotenuse of length \(2x\), a leg of length \(x\), and a leg of length y. Write an expression that shows the value of \(y\) in terms of \(x\).

Since no figure was given, your first step should be to draw one. The order of the legs isn’t important, but remember that the hypotenuse is opposite the right angle.

algebra problem diagram

Now you can apply the Pythagorean theorem to write:

\(x^2 + y^2 = (2x)^2\)

Squaring the right-hand side:

\(x^2 + y^2 = 4x^2\)

When the problem says “the value of \(y\)”, it means you must solve for \(y\). Therefore, we will write:

\(y^2 = 4x^2 – x^2\)

Combining like terms:

\(y^2 = 3x^2\)

Now, use the square root to write:

\(y = \sqrt{3x^2}\)

Finally, this simplifies to give us the expression we are looking for:

\(y = \bbox[border: 1px solid black; padding: 2px]{x\sqrt{3x}}\)

[adsenseLargeRectangle]

The Pythagorean theorem allows you to find the length of any of the three sides of a right triangle. It is one of those things that you should memorize, as it comes up in all areas of math, and therefore in many different math courses you will probably take. Remember to avoid the common mistake of mixing up where the legs go in the formula vs. the hypotenuse and to always draw a picture when one isn’t given.

Share this:

  • Click to share on Twitter (Opens in new window)
  • Click to share on Facebook (Opens in new window)

Cambridge University Faculty of Mathematics

Or search by topic

Number and algebra

  • The Number System and Place Value
  • Calculations and Numerical Methods
  • Fractions, Decimals, Percentages, Ratio and Proportion
  • Properties of Numbers
  • Patterns, Sequences and Structure
  • Algebraic expressions, equations and formulae
  • Coordinates, Functions and Graphs

Geometry and measure

  • Angles, Polygons, and Geometrical Proof
  • 3D Geometry, Shape and Space
  • Measuring and calculating with units
  • Transformations and constructions
  • Pythagoras and Trigonometry
  • Vectors and Matrices

Probability and statistics

  • Handling, Processing and Representing Data
  • Probability

Working mathematically

  • Thinking mathematically
  • Mathematical mindsets
  • Cross-curricular contexts
  • Physical and digital manipulatives

For younger learners

  • Early Years Foundation Stage

Advanced mathematics

  • Decision Mathematics and Combinatorics
  • Advanced Probability and Statistics

Pythagoras' Theorem and Trigonometry - Short Problems

Pythagoras Stage 4 ★            Pythagoras Stage 4 ★★★
             
Pythagoras Stage 4 ★★   Trigonometry Stage 4 ★★★
         
             

pythagorean theorem problem solving questions

A parallelogram is formed by joining together four equilateral triangles. What is the length of the longest diagonal?

Right-angled Midpoints

If the midpoints of the sides of a right angled triangle are joined, what is the perimeter of this new triangle?

Right Angled Possibilities

If two of the sides of a right-angled triangle are 5cm and 6cm long, how many possibilities are there for the length of the third side?

Rectangle Rearrangement

A 3x8 rectangle is cut into two pieces... then rearranged to form a right-angled triangle. What is the perimeter of the triangle formed?

Tetromino Diagonal

Can you calculate the length of this diagonal line?

Pythagoras' Dream

Can you work out the area of this isosceles right angled triangle?

Out of the Window

Find out how many pieces of hardboard of differing sizes can fit through a rectangular window.

Three Right Angles

Work your way through these right-angled triangles to find $x$.

Folded Over

A rectangular piece of paper is folded. Can you work out one of the lengths in the diagram?

Hexagon Perimeter

A circle of radius 1 is inscribed in a regular hexagon. What is the perimeter of the hexagon?

Walk the Plank

A rectangular plank fits neatly inside a square frame when placed diagonally. What is the length of the plank?

Folding in Half

How does the perimeter change when we fold this isosceles triangle in half?

Building Blocks

Can you find the length of AB in this diagram?

Two arcs are drawn in a right-angled triangle as shown. What is the length $r$?

Unusual Polygon

What is the perimeter of this unusually shaped polygon...

Unusual Quadrilateral

This quadrilateral has an unusual shape. Are you able to find its area?

Winding Vine

A vine is growing up a pole. Can you find its length?

Snapped Palm Tree

A palm tree has snapped in a storm. What is the height of the piece that is still standing?

Symmetric Angles

This diagram has symmetry of order four. Can you use different geometric properties to find a particular length?

Strike a Chord

Can you work out the radius of a circle from some information about a chord?

Question of Three Sides

Can you find the length of the third side of this triangle?

Triangular Teaser

Triangle T has sides of lengths 6, 5 and 5. Triangle U has sides of lengths 8, 5 and 5. What is the ratio of their areas?

Diagonal Area

A square has area 72 cm$^2$. Find the length of its diagonal.

Circle Time

Three circles of different radii each touch the other two. What can you deduce about the arc length between these points?

Triple Pythagoras

Can you work out the length of the diagonal of the cuboid?

Interior Squares

Calculate the ratio of areas of these squares which are inscribed inside a semi-circle and a circle.

Height of the Tower

How do these measurements enable you to find the height of this tower?

Centre Square

What does Pythagoras' Theorem tell you about the radius of these circles?

Integers on a Sphere

Can you find all the integer coordinates on a sphere of radius 3?

Oh So Circular

The diagram shows two circles and four equal semi-circular arcs. The area of the inner shaded circle is 1. What is the area of the outer circle?

Diamond Ring

Find the radius of the stone in this ring.

Salt's Mill

A window frame in Salt's Mill consists of two equal semicircles and a circle inside a large semicircle. What is the radius of the circle?

Smartphone Screen

Can you find the length and width of the screen of this smartphone in inches?

Ice Cream Tangent

The diagram shows a semi-circle and an isosceles triangle which have equal areas. What is the value of tan x?

Distance to the Corner

Can you find the distance from the well to the fourth corner, given the distance from the well to the first three corners?

Overlapping Ribbons

Two ribbons are laid over each other so that they cross. Can you find the area of the overlap?

Square Overlap

The top square has been rotated so that the squares meet at a 60$^\text{o}$ angle. What is the area of the overlap?

Common Tangent

Two circles touch, what is the length of the line that is a tangent to both circles?

Folded Rectangle

Can you find the perimeter of the pentagon formed when this rectangle of paper is folded?

Indigo Interior

The diagram shows 8 shaded squares inside a circle. What is the shaded area?

Triangle Radius

Can you find the radii of the small circles?

The Roller and the Triangle

How much of the inside of this triangular prism can Clare paint using a cylindrical roller?

Semicircle in a Semicircle

The diagram shows two semicircular arcs... What is the diameter of the shaded region?

When the Boat Comes In

When you pull a boat in using a rope, does the boat move more quickly, more slowly, or at the same speed as you?

The diagrams show squares placed inside semicircles. What is the ratio of the shaded areas?

Four Circles

Can you find the radius of the larger circle in the diagram?

  • TemplateLab

Pythagorean Theorem Worksheet

48 pythagorean theorem worksheet with answers [word + pdf].

The simplicity of the Pythagorean Theorem worksheet is the best thing about it. What is the Pythagorean Theorem? Formulated in the 6th Century BC by Greek Philosopher and mathematician Pythagoras of Samos, Pythagorean Theorem is a mathematic equation used for a variety of purposes. Over the years, many engineers and architects have used Pythagorean Theorem worksheet to complete their projects .

Table of Contents

  • 1 Pythagorean Theorem Worksheet
  • 2 Knowing Pythagoras of Samos and how he came up with the Pythagorean equation
  • 3 Understanding Pythagorean Theorem
  • 4 Pythagorean Theorem Word Problems Worksheet
  • 5 Using Pythagorean Theorem worksheet
  • 6 Conclusion

A simple equation, Pythagorean Theorem states that the square of the hypotenuse (the side opposite to the right angle triangle) is equal to the sum of the other two sides . Following is how the Pythagorean equation is written:

In the aforementioned equation, c is the length of the hypotenuse while the length of the other two sides of the triangle are represented by b and a. Though the knowledge of the Pythagorean Theorem predates the Greek Philosopher, Pythagoras is generally credited for bringing the equation to the fore. This is the reason the Pythagorean equation is named after him. Before we discuss the Pythagorean Theorem and the Pythagorean Theorem worksheet in detail, let’s take a look at who Pythagoras of Samos was and how he came up with the Pythagorean equation.

Free pythagorean theorem 01

Knowing Pythagoras of Samos and how he came up with the Pythagorean equation

A 6 th century BC Greek philosopher and mathematician, Pythagoras of Samos is widely credited for bringing the Pythagorean equation to the fore. Though others used the relationship long before his time, Pythagoras is the first one who made the relationship between the lengths of the sides on a right-angled triangle public. This is why he’s regarded as the inventor of the Pythagorean equation.

Apart from being a philosopher and mathematician, Pythagoras founded the Pythagoreanism movement. Born in Croton, Italy, Pythagoras travelled to many different countries including Greece, Egypt, and India. After moving back to Croton in 530 BC, Pythagoras established some kind of school. He returned to Samos in 520 BC. It was in late 6 th Century BC that Pythagoras started to make important contributions to philosophy and math. The Pythagorean equation was one of those contributions.

Though he revealed the Pythagorean equation to the world in the late 6 th Century BC while living in Samos, many historians believe that Pythagoras first thought about the equation during his time in Egypt. In fact, according to many historians, Pythagoras learned geometry, the Phoenicians arithmetic and other branches of mathematics from the Egyptians.

Though he has made many important contributions to philosophy, Pythagoras is widely known as the founder of the Pythagorean Theorem. As previously mentioned, the Pythagorean Theorem is a mathematical equation that states that the square of the hypotenuse (the side opposite to the right angle triangle) is equal to the sum of the other two sides .

Today the aforementioned equation bears Pythagoras’s name but it’s important to know that he wasn’t the first one to use the equation. Before Pythagoras’s time, the Indians and the Babylonians utilized the Pythagorean Theorem or equation. Since they constructed the first proof of the theorem, Pythagoras and his disciples are regarded as the inventors of the equation.

Many historians say that Pythagoras worked in a very secretive manner. This is the reason little evidence is available that the Greek Philosopher/ mathematician himself worked on and proved the Pythagorean Theorem. It is important to note that the first time Pythagoras was given credit for the Theorem was five centuries after his death. This makes Pythagoras’s contribution to the Theorem even more debatable. Nonetheless, since Pythagoras is the only one connected to the Pythagorean Theorem known today, we have to give him due credit. Now that we’ve discussed who Pythagoras of Samos was and how he came up with the Pythagorean equation, it’s time to take a detailed look at the Pythagorean Theorem and the Pythagorean Theorem worksheet.

Free pythagorean theorem 13

Understanding Pythagorean Theorem

According to Pythagorean Theorem, the sum of the squares on the right-angled triangle’s two smaller sides is equal to the side opposite to the right angle triangle (the square on hypotenuse). Using a Pythagorean Theorem worksheet is a good way to prove the aforementioned equation. An amazing discovery about triangles made over two thousand years ago, Pythagorean Theorem says that when a triangle has a 90° angle and squares are made on each of the triangle’s three sides, the size of the biggest square is equal to the size of the other two squares put together! A short equation, Pythagorean Theorem can be written in the following manner:

In Pythagorean Theorem, c is the triangle’s longest side while b and a make up the other two sides. The longest side of the triangle in the Pythagorean Theorem is referred to as the ‘hypotenuse’. Many people ask why Pythagorean Theorem is important. The answer to this is simple: you’ll be able to find the length of a right-angled triangle’s third side if you know the length of the other two sides. This equation works like magic and can be used to find any missing value. Following is an example that uses the Pythagorean Theorem to solve a triangle.

In this equation, the longest side of the triangle ‘c’ is missing. By finding out the sum of the squares of the two other sides, we were able to find the missing value. The most famous mathematical contribution of Pythagoras, the Pythagoras Theorem was one of the earliest documented theorems. Though Pythagoras is given most of the credit for the theorem, a major contribution to the theorem was made by his students.

When you look at a Pythagoras Theorem worksheet, you’ll notice that the theorem enables you to find the length of any right angle triangle side provided you know the length of the other two sides. Also, using the theorem, you can check whether a triangle is a right triangle. The Pythagoras Theorem is extremely useful in solving many math problems. Further, you can use it in many real life situations. This is illustrated by a Pythagoras Theorem worksheet.

Pythagorean Theorem Word Problems Worksheet

Free pythagorean theorem 22

Using Pythagorean Theorem worksheet

A good way to review the Pythagoras Theorem and expand the mathematical equation is using a Pythagoras Theorem worksheet. By using the worksheet, you’ll be able to get a good understanding of geometry. Additionally, the worksheet will give you an opportunity to review the knowledge related to the different types of triangles. Finally and most importantly, you’ll be able to practice the ancient equation invented by the Greek mathematician and philosopher, Pythagoras. Before you start using the Pythagoras Theorem worksheet, just remember that ‘c’ is the hypotenuse while the shorter sides of the triangle are represented by ‘a’ and ‘b’.

A Pythagoras Theorem worksheet presents students with triangles of various orientations and asks them to identify the longest side of the triangle i.e. the hypotenuse. As you know by now, the formula used in Pythagoras Theorem is a²+b²=c². Regardless of what the worksheet asks the students to identify, the formula or equation of the theorem always remain the same. Though, the students could be presented with different challenges including solving triangles:

  • Labeled in different order
  • With a different set of letters
  • By using vertices to name the sides

The symbols used in the Pythagoras Theorem are something students will find on their calculators. Figuring out how to use these functions is what students need to establish. There is involvement of the Babylonians and the Egyptians in the invention of the Pythagoras Theorem but the earliest known proof of the theorem was produced by the school of Pythagoras.

Many Pythagorean triples were known to the Babylonians while the Egyptians knew and used the (3, 4, 5) triple. The Chinese and Indians also played a role in the invention of the Pythagoras Theorem. The first diagrammatic proof of the theorem was produced by the Chinese while the Indians discovered many triples. In 1995, the theorem became part of the Guinness Book of Records as the most proved theorem of all time.

The triples used in the Pythagoras Theorem include (3, 4, 5), (6, 8, 10), (5, 12, 13), (8,15,17), (7,24,25), (20,21,29), (12,35,37), (9,40,41), (28,45,53), (11,60,61), (16,63,65), (33,56,65) and (48,55,73). The aforementioned triples aren’t multiples of a smaller triple and the name given to them is ‘primitive’ triples. To solve a particular problem, the Pythagoras Theorem can be arranged. For example, if you’re asked to find b which is one of the two smaller sides of the right-angled triangle, you can rearrange the theorem to b²=c²-a². By doing this, you’ll be able to easily find the missing value.

The Pythagoras Theorem has many different proofs. However, when checking your answers, following are the two things that you must always remember:

  • The side opposite to the right angle or simply the hypotenuse is always the longest side of the triangle
  • Though it is the longest side of the triangle, the size of the hypotenuse can never exceed the sum of the other two squares

To understand this better, take a look at a Pythagoras Theorem worksheet. Today, you can get easy access to Pythagorean Theorem worksheet with answers. Nonetheless, we’re going to try and understand the Pythagoras Theorem as much as we can.

As mentioned earlier, if you know the size of the other two sides, you will be able to find out the length of the third side of the right angle triangle. Also, after being squared, the shorter length is subtracted from the square of the hypotenuse when the hypotenuse is one of the two known lengths. As seen earlier, the lengths of each side of the triangle in the Pythagoras Theorem are whole numbers. Such triangles are known as Pythagorean triangles.

Though there are many different proofs of the Pythagoras Theorem, only three of them can be constructed by students and other people on their own. The first proof starts off as rectangle and is then divided into three triangles that individually contain a right angle. To see first proof, you can use a computer or something as straight forward as an index card cut up into right triangles.

Beginning with a rectangle, the second proof of the Pythagoras Theorem starts off by constructing rectangle CADE with BA=DA. This is followed by the construction of the <BAD’s angle bisector. Once constructed, the bisector is allowed to intersect ED at point F. This makes <BAF and <DAF congruent, BA=DA, and AF=AF. This in turn makes the triangle DAF equal to triangle BAF which means that since ADF is a right angle, ABF will also be a right angle. The third and final proof of the Pythagorean Theorem that we’re going to discuss is the proof that starts off with a right angle. In this proof, triangle ABC is right angle and its right side is angle C.

The three proofs stated above are just few of the many Pythagoras Theorem. You’ll come across these proofs when you take a look at the Pythagorean Theorem worksheet with answers. Learning and understanding the Pythagorean concept is extremely important for students and other people who’ll use this theorem in their practical life.

It is important that you understand the algebraic representation of the Pythagoras Theorem as well as the geometric concepts behind it. You can accomplish this by using proofs, manipulatives, and computer technology. By using these methods to learn Pythagorean Theorem, you’ll be able to see the connections and benefit greatly.

Free pythagorean theorem 32

Formulated in the 6th Century BC by Pythagoras of Samos, Pythagoras Theorem is widely used today. If you want to practice Pythagoras Theorem then you can do that easily. Pythagoras Theorem worksheets with answers are easily available and you can use these worksheets to get a good grip of the Theorem.

More Templates

Law School Letters Of Recommendation

Law School Letters Of Recommendation

Community Service Forms

Community Service Forms

Genogram Templates

Genogram Templates

Permission Slip Templates

Permission Slip Templates

Story Map Templates

Story Map Templates

Unit Circle Charts

Unit Circle Charts

Pythagorean theorem - practice problems

Number of problems found: 1366.

' width=

  • all math problems 19043
  • planimetrics 3456
  • Pythagorean theorem 1366
  • right triangle 1035
  • triangle 904
  • expression of a variable from the formula 549
  • square root 433
  • area of a shape 418
  • diagonal 302
  • surface area 277

logo of hackmath

  • New math problems
  • Popular math problems
  • Harder math problems
  • The easiest word problems

REAL WORLD PROBLEMS ON PYTHAGOREAN THEOREM

Problem 1 :

A man goes 18 m due east and then 24 m due north. Find the distance of his current position from the starting point?

pythagorean theorem problem solving questions

the distance of his current position from the starting point  =   √18 2 + 24 2

  =   √(324 + 576)

  =   √900

  =  30 m

So, the required distance is 30 m.

Problem 2 :

There are two paths that one can choose to go from Sarah’s house to James house. One way is to take C street, and the other way requires to take A street and then B street. How much shorter is the direct path along C street? (Using figure).

pythagorean theorem problem solving questions

By choosing the C street, he has to cover the distance,

 =   √2 2  + 1.5 2

 =   √(4 + 2.25)

 =   √6.25

 =  2.5 miles

By choosing the alternative way, he has to cover the distance  =  2 + 1.5

=  3.5 miles

The difference between these two paths =  3.5 - 2.5

  =  1 mile

So, by choosing the direct path, he may save 1 miles faster than other way.

Problem 3 :

To get from point A to point B you must avoid walking through a pond. You must walk 34 m south and 41 m east. To the nearest meter, how many meters would be saved if it were possible to make a way through the pond?

By drawing the rough picture using the given information, we get 

pythagorean theorem problem solving questions

AC  =   √34 2  + 41 2

   =   √1156 + 1681

  =   √2837

  =  53.26

Miles saved  =  (34 + 41) - 53.26

  =  75 - 53.26

=  21.74 m

Problem 4 :

In the rectangle WXYZ, XY + YZ = 17 cm, and XZ + YW = 26 cm. Calculate the length and breadth of the rectangle?

pythagorean theorem problem solving questions

XY + YZ = 17 cm

XZ + YW = 26 cm

To calculate : - Length and breadth of the rectangle.

We know that,

Diagonals of a rectangle are equal.

XZ = YW = 26/2 = 13 cm

In ∆XYZ, let YZ = P. Then

XY = 17 - P

Then, by Pythagoras theorem,

(P) 2 + (17 - P) 2 = (13) 2

P 2  + 289 - 34P + P 2  = 169

2P 2  - 34P = 169 - 289

2(P 2  - 17P) = - 120

 P 2  - 17P = - 120/2

P 2  - 17P = - 60

P 2  - 17P + 60 = 0

P 2  - 12P - 5P + 60 = 0

 P(P - 12) - 5(P - 12) = 0

(P - 12)(P - 5) = 0

P - 12  =  0  or  P  =  12

P  =  12 cm  or  P  =  5 cm

YZ = P = 12 cm [Because , YZ is the length of the rectangle ,so we will assign it the greatest value of P]

Again, XY = (17 - P) = (17 - 12) cm = 5 cm

[Because , XY is thee breadth]

Kindly mail your feedback to   [email protected]

We always appreciate your feedback.

© All rights reserved. onlinemath4all.com

  • Sat Math Practice
  • SAT Math Worksheets
  • PEMDAS Rule
  • BODMAS rule
  • GEMDAS Order of Operations
  • Math Calculators
  • Transformations of Functions
  • Order of rotational symmetry
  • Lines of symmetry
  • Compound Angles
  • Quantitative Aptitude Tricks
  • Trigonometric ratio table
  • Word Problems
  • Times Table Shortcuts
  • 10th CBSE solution
  • PSAT Math Preparation
  • Privacy Policy
  • Laws of Exponents

Recent Articles

Sat math resources (videos, concepts, worksheets and more).

Aug 02, 24 11:32 AM

Pre Calculus Problems and Solutions (Part - 7)

precalculus3.png

Digital SAT Math Problems and Solutions (Part - 18)

Aug 02, 24 11:23 AM

Ollie Lovell

Username or email address  *

Password  *

Remember me Log in

Lost your password?

No products in the cart.

Pythagoras’ Theorem: My attempt to write challenging questions and engage students

On my last placement I taught a unit on Pythagoras' theorem to my year 9 class. I found it deceptively difficult. Why? The challenge was twofold: First, students had seen the theorem in previous years so it was a bit old hat to them. Second, I wondered how to stretch students and provide them with variety and challenge beyond the old trick of calling the hypotenuse a ladder?

I was thinking to myself ‘ WHY does Pythagoras’ theorem actually matter, and how can I make that tangible and relevant to students? I came up with the following slideshow.

I had this first slide on the screen as students walked into the classroom, it created a bit of a buzz in the room…

I got students to vote (hands up) on which corner they thought was square (which do you think it is?), then revealed the answer.

I've presented the remainder of the slideshow in the following video with examples of the kind of teacher questions and the script that I used. Seemed easier and clearer than writing it all out…

(If you want to use any of this you can download the slideshow file here:  Pythagoras Slides_Oliver Lovell_www.ollielovell.com )

Clarifying points…

  • I used the qualifier of ‘on the macro scale' when talking about ‘no squares in nature' because I thought it's probably possible for there to be perfect squares in crystalline atomic structures or something like that.
  • Perhaps an oversight in the video is a discussion of the meaning of the word ‘breadth' (I've become more aware of the vocab that I'm using in class since my resent teaching placement in Myanmar where the students had very limited english. Post to come).

The general structure of lessons in this unit from then on was a bit of book work (using the ‘points' system that I'll write about in another post) with a challenge question for those students who got all of their points done. Lessons were wrapped up with students sharing on the board their approach to the challenge question and what they had learned from attempting it. I also book-ended most lessons with a micro-revision, 3 questions on the basics as a mini-test at the start, and an exit card at the end to help me to keep track of how students were progressing.

Below are the questions that I used as challenge questions. I wrote all of the following questions, excepting the final question, which I found here . I'd also like to acknowledge Dr Max Stephens who suggested that I create questions that encourage students to solve for exact values rather than untidy decimals. I've left out answers on purpose in the knowledge that future students will possibly visit this post.

Pythagoras Extension Questions:

You’ve always got to include a ladder question don’t you? (this one is a little bit different).

A Cat is stuck in a tall tree and needs to be saved by the Firefighters. The fire truck has a 35m ladder. The closest that it can get to the tree (because there are parked cars in the way) is 12m. The cat is in the tree, 39m up. Comment on the likelihood of the cat getting saved and justify your suggestion mathematically.

Q:  A cube has side length of 10cm. what is the length of the diagonal through its centre? (could accompany this with the picture below).

Alternatively you could ask, Q:  ‘What is the length of d in the following if the side lengths are 10cm?' (referring to the above image)

Prior to giving students the above question we’d had a discussion about the formula of the diagonal of a box and students had worked out that it’s just the square root of the sum of the squares of the box’s dimensions. This helped them to solve this question. One of the students actually proposed this ‘pythagoras in 3 dimensions' formula, so I encouraged them all to check it and see if the proposal seemed plausible.

An additional question that I also posed to students was,  Q:  “Show that the formula for the diagonal of a box is √a²+b²+c².” A discussion of this also helped to scaffold the students.

Q:  A rectangular prism has sides of length a, 4a/3, and 4a. It has an internal diagonal of 26cm. What are the dimensions of the rectangular prism?

Or, you could word it in a more tricky way:   Q:  A rectangular prism has a width one quarter of its length and a height one third of its width. It has an internal diagonal of 26cm. What are the dimensions of the rectangular prism?

Some other interesting shapes…

Another question:

In the 6 sided pyramid pictured below, what is the height of the apex ‘V’ above the point  ‘O’ ? (You can assume that O is in the centre of the base, the base is a regular hexagon, and V is directly above O).

Resources to run a PD on ‘Hinge Questions’: Based on Dylan Wiliam’s ‘ACER Teacher’ podcast

Hinge Question: -A question that decides whether the students have understanding of the topic before you move on -A question…

Interthinking: How to make group work, work

I just had one of the most enjoyable classes that I’ve taught in a long time. It was focused on group…

Communicating a schema: How do I get it from my brain into my students’?

Background: Research tells us that when helping students to learn new concepts it's important to support them to draw connections between what they're…

Implementation Intentions and action triggers: moving beyond ‘Well, make sure you do it next time’

A student has forgotten something, whether it be their homework, to bring back a form, to bring their charged up…

  • School Guide
  • Mathematics
  • Number System and Arithmetic
  • Trigonometry
  • Probability
  • Mensuration
  • Maths Formulas
  • Class 8 Maths Notes
  • Class 9 Maths Notes
  • Class 10 Maths Notes
  • Class 11 Maths Notes
  • Class 12 Maths Notes

Pythagorean Identities Practice Problems

Pythagorean identities are fundamental trigonometric identities derived from the Pythagorean theorem. They establish essential relationships between the sine, cosine, and tangent functions. These identities are crucial in simplifying complex trigonometric expressions and solving various mathematical problems.

In this article, we will learn what are Pythagorean identities, derivations of other identities some more important trigonometry identities, and some practice questions on Pythagorean identities.

What is Pythagorean Identities

Pythagorean identities are composed of these six trigonometric functions present in mathematics: sine, cosine, tan, sec, cosec, and cot. The trigonometric functions sine (sin), cosine (cos), and tangent (tan) are involved in the three main Pythagorean identities , which are obtained from the Pythagorean theses. These identities are:

sin²θ + cos²θ = 1 1 + tan²θ = sec²θ 1 + cot²θ = cosec²θ

Problem 1: Verify the identity: sin 2 θ + cos 2 θ = 1 for θ = 45 o .

Sin45 o = √2/2 cos 45 o = √2/2 sin 2 45 o + cos 2 45 o = (√2/2) 2 + (√2/2) 2 = 2/4 + 2/4 = 4/4 = 1

Problem 2: If cosθ = 5/13 , then find sinθ.

Using the identity: sin 2 θ+ cos 2 θ = 1 sin 2 θ + (5/13) 2 = 1 sin 2 θ + 25/169 = 1 sin 2 θ = 1 – 25/169 sin 2 θ = 144/169 sinθ = ± 12/13

Problem 3: Solve for θ if sec 2 θ = 5.

Using the identity: sec 2 θ = 1 + tan 2 θ 5 = 1 + tan 2 θ tan 2 θ = 4 tanθ= ±2 θ = tan -1 (2)

Problem 4: Prove the identity: cos 2 θ = 1 – sin 2 θ.

Using the basic identity: sin 2 θ + cos 2 θ = 1 cos 2 θ = 1 – sin 2 θ

Problem 5: If sinθ = 3/5 , then find the value of cos θ using Pythagorean identity.

Using the identity: sin 2 θ+ cos 2 θ = 1 (3/5) 2 + cos 2 θ = 1 9/25 + cos 2 θ = 1 cos 2 θ = 1 – 9/25 cos 2 θ = 16/25 cosθ = ± 4/5

Problem 6: Find cosec θ if cot θ = 2.

Using the identity: cosec 2 θ – cot 2 θ = 1 1 + cot 2 θ = cosec 2 θ 1 + 2 2 = cosec 2 θ 1 + 4 = cosec 2 θ cosec 2 θ = 5 cosecθ = ±√5

Problem 7: Express cot 2 θ in terms of cosec 2 θ.

We know by identity that: 1 + cot 2 θ = cosec 2 θ Rearranging above equation we get: cot 2 θ = cosec 2 θ – 1

Problem 8: Find sinθ if secθ = 2.

secθ = 1/cosθ secθ = 2 cosθ = 1/2 sin 2 θ + cos 2 θ = 1 sin 2 θ + (1/2) 2 = 1 sin 2 θ= 1 – 1/4 sin 2 θ = 3/4 sinθ = ± √3/2

Problem 9: If sinθ = 1/2, find tanθ.

Using the identity: sin 2 θ + cos 2 θ = 1 (1/2) 2 + cos 2 θ = 1 cos 2 θ = 1 – 1/4 cos 2 θ = 3/4 cosθ = √3/2 tanθ = sinθ/cosθ tanθ = 1/2/√3/2 tanθ = 1/√3

Problem 10: Simplify the expression using Pythagorean identities: sin 2 θ + cos 2 θ + 1.

By Pythagorean identity, we know that: sin 2 θ + cos 2 θ = 1 So, the value of sin 2 θ + cos 2 θ + 1 = 1 + 1 = 2

Practice Questions on Pythagorean Identities

Practice-Questions-on-PI-001-----Google-Docs_page-0001

You can download these Practice problems and their Solution from – Pythagorean Identities Practice Problems

Pythagorean identities, which in real life connect the basic trigonometric functions, are an essential part of trigonometry. It defines all three main formulas of trigonometry. After understanding these Pythagorean identities one has a deeper understanding of trigonometric identities and will be able to solve questions. The identities and practice questions given in this article will help you to understand it in a better way.

Related Articles:

Pythagorean Identities Trigonometric Identities Trigonometric Identities Class 10

FAQs on Pythagorean Identities

What are pythagorean identities.

Trigonometric identities also known as Pythagorean identities, which express relationships between the sine, cosine, and tangent functions, are derived using the famous Pythagorean theorem.

Why are Pythagorean identities important?

They are important for deriving solutions to trigonometric equations and serve as the basic understanding for different calculus and physics ideas.

How are Pythagorean identities derived?

They are derived by applying the Pythagorean theorem to a right triangle and then changing the fundamental identities(by exhanging LHS and RHS side of equation) to obtain other identities.

Is it possible to use Pythagorean identities in Calculus?

Yes, they are important for solving a variety of calculus problems, such as those involving the differentiation and integration of trigonometric functions.

What differentiate the derived Pythagorean identities from the basic ones?

The main relationships that are directly derived from the Pythagorean theorem are known as basic identities, and these basic relationships can be changed to create derived identities.

Please Login to comment...

Similar reads.

  • School Learning
  • Practice Questions

Improve your Coding Skills with Practice

 alt=

What kind of Experience do you want to share?

Mathwarehouse Logo

Right Triangles

Rules, Formula and more

Pythagorean Theorem

The sum of the squares of the lengths of the legs equals the square of the length of the hypotenuse .

Usually, this theorem is expressed as $$ A^2 + B^2 = C^2 $$ .

Right Triangle Properties

Right triangle picture

A right triangle has one $$ 90^{\circ} $$ angle ($$ \angle $$ B in the picture on the left) and a variety of often-studied formulas such as:

  • The Pythagorean Theorem
  • Trigonometry Ratios (SOHCAHTOA)
  • Pythagorean Theorem vs Sohcahtoa (which to use)

SOHCAHTOA only applies to right triangles ( more here ) .

sohcahtoa

A Right Triangle's Hypotenuse

The hypotenuse is the largest side in a right triangle and is always opposite the right angle.

Hypotenuse

In the triangle above, the hypotenuse is the side AB which is opposite the right angle, $$ \angle C $$.

Online tool calculates the hypotenuse (or a leg) using the Pythagorean theorem.

Practice Problems

Below are several practice problems involving the Pythagorean theorem, you can also get more detailed lesson on how to use the Pythagorean theorem here .

Find the length of side t in the triangle on the left.

5, 12, 13 right triangle

Substitute the two known sides into the Pythagorean theorem's formula : A² + B² = C²

What is the value of x in the picture on the left?

pythagorean theorem

Set up the Pythagorean Theorem : 14 2 + 48 2 = x 2 2,500 = X 2

$$ x = \sqrt{2500} = 50 $$

Diagram, Pythagorean Theorem

$$ x^2 = 21^2 + 72^2 \\ x^2= 5625 \\ x = \sqrt{5625} \\ x =75 $$

Find the length of side X in the triangle on on the left?

3, 4, 5 right triangle

Substitue the two known sides into the pythagorean theorem's formula : $$ A^2 + B^2 = C^2 \\ 8^2 + 6^2 = x^2 \\ x = \sqrt{100}=10 $$

What is x in the triangle on the left?

pythagorean image

x 2 + 4 2 = 5 2 x 2 + 16 = 25 x 2 = 25 - 16 = 9 x = 3

Ultimate Math Solver (Free) Free Algebra Solver ... type anything in there!

Popular pages @ mathwarehouse.com.

Surface area of a Cylinder

IMAGES

  1. Pythagorean Theorem Worksheets

    pythagorean theorem problem solving questions

  2. Question Video: Applying the Pythagorean Theorem to Solve Problems

    pythagorean theorem problem solving questions

  3. 😀 Pythagorean theorem problem solving. Tenth grade Lesson Using the

    pythagorean theorem problem solving questions

  4. Pythagorean Theorem Worksheet With Answers

    pythagorean theorem problem solving questions

  5. Pythagoras Theorem

    pythagorean theorem problem solving questions

  6. 4 Ways to Solve Pythagoras Theorem Questions

    pythagorean theorem problem solving questions

VIDEO

  1. Pythagorean Theorem Two Proofs

  2. The Pythagorean Theorem

  3. The Pythagorean Theorem (Astraveo Education Series)

  4. Pythagorean Theorem and a Quilt

  5. Applications of the Pythagorean Theorem

  6. Pythagorean Theorem word problems-Geometry Help

COMMENTS

  1. Pythagorean Theorem Practice Problems With Answers

    Here's the Pythagorean Theorem formula for your quick reference. The longer leg is twice the shorter leg. Find the hypotenuse. If the longest leg is half the hypotenuse, what is the length of the shortest leg? Here are eight (8) Pythagorean Theorem problems for you to solve. You might need to find either the leg or the hypotenuse of the right ...

  2. 15 Pythagorean Theorem Practice Problems For 8th Grade

    How to answer Pythagorean Theorem questions. 1 - Label the sides of the triangle a, b, and c. Note that the hypotenuse, the longest side of a right triangle, is opposite the right angle and will always be labeled. 2 - Write down the formula and substitute the values>. a^2+b^2=c^2 a2 +b2 = c2. 3 - Calculate the answer.

  3. Pythagoras Theorem Questions [Solved]

    Practice Questions on Pythagoras Theorem. 1. Find the area of a right-angled triangle whose hypotenuse is 13 cm and one of the perpendicular sides is 5 cm. 2. Find the Pythagorean triplet whose one member is 15. 3. Find the perimeter of a rectangle whose diagonal is 5 cm and one of its sides is 4 cm.

  4. PDF Pythagorean Theorem

    a) d) 8) A right triangle has legs of 52.6 cm and 35.7 cm. Determine the length of the triangle's hypotenuse. 9) A right triangle has a hypotenuse of 152.6 m. The length of one of the other sides is 89.4 m. Determine the length of the third side. 10) For each of the following, the side lengths of a triangle are given.

  5. Pythagoras Theorem Questions

    The following questions involve using Pythagoras' theorem to solve a range of word problems involving 'real-life' type questions. On the first sheet, only the hypotenuse needs to be found, given the measurements of the other sides. Illustrations have been provided to support students solving these word problems.

  6. Pythagorean Theorem

    What is the Pythagorean Theorem? The Pythagorean Theorem states that the square of the longest side of a right triangle (called the hypotenuse) is equal to the sum of the squares of the other two sides.. Pythagorean Theorem formula shown with triangle ABC is:. a^2+b^2=c^2 . Side c is known as the hypotenuse.The hypotenuse is the longest side of a right triangle.

  7. Pythagorean Theorem Practice Problems and Examples

    kramberol 2022-07-11. pythagorean theorem extensions. are there for a given integer N solutions to the equations. ∑ n = 1 N x i 2 = z 2. for integers x i and zan easier equation given an integer number 'a' can be there solutions to the equation. ∑ n = 1 N x i 2 = a 2. for N=2 this is pythagorean theorem.

  8. Pythagorean theorem word problems (practice)

    Pythagorean theorem word problems. Google Classroom. Microsoft Teams. You might need: Calculator. Steve is turning half of his backyard into a chicken pen. His backyard is a 24 meter by 45 meter rectangle. He wants to put a chicken wire fence that stretches diagonally from one corner to the opposite corner. How many meters of fencing will Steve ...

  9. Pythagorean theorem challenge (practice)

    Learn for free about math, art, computer programming, economics, physics, chemistry, biology, medicine, finance, history, and more. Khan Academy is a nonprofit with the mission of providing a free, world-class education for anyone, anywhere. ... Multi-step word problem with Pythagorean theorem . Report a problem

  10. PDF Practice Questions (and Answers)

    Pythagorean Theorem: where a and b are lengths of the legs of a fight triangle and c is the length of the hypotenuse "sum of the squares of the legs is equal to the square of the hypotenuse" Example: 49 _ 65 c fight triangle acute triangle obtuse triangle AV Identifying triangles by their sides: a a a Distance Formula mustrates Pythagorean Theorem!

  11. Pythagorean theorem intro problems (article)

    Pythagorean theorem intro problems. ... Practice using the Pythagorean theorem to solve for missing side lengths on right triangles. Each question is slightly more challenging than the previous. Pythagorean theorem. ... Just a quick question, so in a test it could say find the distance of the other 2 sides of the triangle. ...

  12. Pythagoras Practice Questions

    The Corbettmaths Practice Questions on Pythagoras. Next: Direct and Inverse Proportion Practice Questions

  13. Pythagoras Theorem Questions (with Answers)

    Right Triangle Questions - using the theorem. The Theorem helps us in: Finding Sides: If two sides are known, we can find the third side. Determining if a triangle is right-angled: If the sides of a triangle are known and satisfy the Pythagoras Formula, it is a right-angled triangle. There is a proof of this theorem by a US president.

  14. Pythagorean Theorem: Problems with Solutions

    Calculate the total area of the plot. Answer: square kilometers. It is given the length of the diagonal of the square. It divides it into two equal triangles. In addition, the two triangles are right and the legs of the same length. be the length of square side and by the Pythagorean theorem we get: \displaystyle x^ {2} +x^ {2}=\left (2\sqrt {2 ...

  15. Word problems on Pythagorean Theorem

    Word problems using the Pythagorean Theorem: 1. A person has to walk 100 m to go from position X in the north of east direction to the position B and then to the west of Y to reach finally at position Z. The position Z is situated at the north of X and at a distance of 60 m from X. Find the distance between X and Y. Solution:

  16. The Pythagorean theorem with examples

    Solution. The side opposite the right angle is the side labelled \ (x\). This is the hypotenuse. When applying the Pythagorean theorem, this squared is equal to the sum of the other two sides squared. Mathematically, this means: \ (6^2 + 8^2 = x^2\) Which is the same as: \ (100 = x^2\) Therefore, we can write:

  17. Pythagoras' Theorem and Trigonometry

    Pythagoras' Theorem and Trigonometry - Short Problems. This is part of our collection of Short Problems. You may also be interested in our longer problems on Pythagoras' Theorem and Trigonometry. Printable worksheets containing selections of these problems are available here: Pythagoras Stage 4 ★. Sheet 1.

  18. 48 Pythagorean Theorem Worksheet with Answers [Word + PDF]

    A simple equation, Pythagorean Theorem states that the square of the hypotenuse (the side opposite to the right angle triangle) is equal to the sum of the other two sides. Following is how the Pythagorean equation is written: a²+b²=c². In the aforementioned equation, c is the length of the hypotenuse while the length of the other two sides ...

  19. Pythagorean theorem

    Pythagorean theorem - practice problems. The Pythagorean Theorem states that in a right-angled triangle, the square of the length of the hypotenuse (the side opposite the right angle) is equal to the sum of the squares of the lengths of the other two sides. This can be written as: where c is the length of the hypotenuse, and a and b are the ...

  20. How to Use the Pythagorean Theorem. Step By Step Examples and Practice

    Example 2 (solving for a Leg) Use the Pythagorean theorem to determine the length of X. Step 1. Identify the legs and the hypotenuse of the right triangle . The legs have length 24 and X X are the legs. The hypotenuse is 26. Step 2. Substitute values into the formula (remember 'C' is the hypotenuse). A2 + B2 = C2 x2 + 242 = 262 A 2 + B 2 = C 2 ...

  21. Real World Problems on Pythagorean Theorem

    By choosing the alternative way, he has to cover the distance = 2 + 1.5. = 3.5 miles. The difference between these two paths = 3.5 - 2.5. = 1 mile. So, by choosing the direct path, he may save 1 miles faster than other way. Problem 3 : To get from point A to point B you must avoid walking through a pond. You must walk 34 m south and 41 m east.

  22. Pythagoras' Theorem: challenging questions, engage students

    An additional question that I also posed to students was, Q: "Show that the formula for the diagonal of a box is √a²+b²+c².". A discussion of this also helped to scaffold the students. Q: A rectangular prism has sides of length a, 4a/3, and 4a. It has an internal diagonal of 26cm.

  23. Pythagorean Identities Practice Problems

    Pythagorean identities are fundamental trigonometric identities derived from the Pythagorean theorem. They establish essential relationships between the sine, cosine, and tangent functions. ... these Pythagorean identities one has a deeper understanding of trigonometric identities and will be able to solve questions. The identities and practice ...

  24. Right Triangles, Hypotenuse, Pythagorean Theorem Examples and Practice

    A Right Triangle's Hypotenuse. The hypotenuse is the largest side in a right triangle and is always opposite the right angle. (Only right triangles have a hypotenuse ). The other two sides of the triangle, AC and CB are referred to as the 'legs'. In the triangle above, the hypotenuse is the side AB which is opposite the right angle, ∠C ∠ C .

  25. The Pythagorean Theorem

    The Pythagorean Theorem [tex]\bullet\,\,\,\boldsymbol{a^2+b^2=c^2.}\bullet[/tex]. in which a and b are the two legs of the right triangle, and c is the hypotenuse, the longest side of the right triangle. ... The question is incomplete but I will try to help you the much I can. ... Solving Problems Involving Inverse Functions With solutions ...

  26. Artificial Intelligence Has a Math Problem

    A.I.'s math problem reflects how much the new technology is a break with computing's past. By Steve Lohr In the school year that ended recently, one class of learners stood out as a seeming ...